cat 1

39
MBA ENTRANCE EXAMINATION AT A GLANCE 1 Directions (Q Directions (Q Directions (Q Directions (Q Directions (Q s. 1 to 5) : s. 1 to 5) : s. 1 to 5) : s. 1 to 5) : s. 1 to 5) : Sentences Sentences Sentences Sentences Sentences given in each question, when properly given in each question, when properly given in each question, when properly given in each question, when properly given in each question, when properly sequenced, for sequenced, for sequenced, for sequenced, for sequenced, form a coher m a coher m a coher m a coher m a coherent paragraph. ent paragraph. ent paragraph. ent paragraph. ent paragraph. Each sentence is labelled with a letter Each sentence is labelled with a letter Each sentence is labelled with a letter Each sentence is labelled with a letter Each sentence is labelled with a letter. Choose the most logical order of these Choose the most logical order of these Choose the most logical order of these Choose the most logical order of these Choose the most logical order of these sentences from among the four given sentences from among the four given sentences from among the four given sentences from among the four given sentences from among the four given alter alter alter alter alternatives to constr natives to constr natives to constr natives to constr natives to construct a coher uct a coher uct a coher uct a coher uct a coherent ent ent ent ent paragraph. paragraph. paragraph. paragraph. paragraph. 1. 1. 1. 1. 1. A. In rejecting the functionalism in positivist organisation theory, either wholly or partially, there is often a move towards a political model of organisation theory. B. Thus the analysis would shift to the power resources possessed by different groups in the organisation and the way they use these resources in actual power plays to shape the organisational structure. C. At the extreme, in one set of writings, the growth of administrators in the organisation is held to be completely unrelated to the work to be done and to be caused totally by the political pursuit of self-interest. D. The political model holds that individual interests are pursued Section I in organisational life through the exercise of power and influence. (1) ADBC (2) CBAD (3) DBCA (4) ABDC 2. 2. 2. 2. 2. A. Group decision making, however, does not necessarily fully guard against arbitrari- ness and anarchy, for individual capriciousness can get substituted by collusion of group members. B. Nature itself is an intricate system of checks and balances, meant to preserve the delicate balance between various environmental factors that affect our ecology. C. In institutions also, there is a need to have in place a system of checks and balances which inhibits the concentration of power in only some individuals. D. When human interventions alter this delicate balance, the outcomes have been seen to be disastrous. (1) CDAB (2) BCAD (3) CABD (4) BDCA Sample Test Questions from IIMs CAT Bulletin for December 2000 Exam For the Session beginning July 2001

Upload: sharmili-priyadarsini

Post on 11-May-2015

695 views

Category:

Technology


0 download

TRANSCRIPT

Page 1: Cat 1

MBA ENTRANCE EXAMINATION AT A GLANCE 1

Directions (QDirections (QDirections (QDirections (QDirections (Qs. 1 to 5) : s. 1 to 5) : s. 1 to 5) : s. 1 to 5) : s. 1 to 5) : SentencesSentencesSentencesSentencesSentencesgiven in each question, when properlygiven in each question, when properlygiven in each question, when properlygiven in each question, when properlygiven in each question, when properlysequenced, forsequenced, forsequenced, forsequenced, forsequenced, form a coherm a coherm a coherm a coherm a coherent paragraph.ent paragraph.ent paragraph.ent paragraph.ent paragraph.Each sentence is labelled with a letterEach sentence is labelled with a letterEach sentence is labelled with a letterEach sentence is labelled with a letterEach sentence is labelled with a letter.....Choose the most logical order of theseChoose the most logical order of theseChoose the most logical order of theseChoose the most logical order of theseChoose the most logical order of thesesentences from among the four givensentences from among the four givensentences from among the four givensentences from among the four givensentences from among the four givenalteralteralteralteralternatives to constrnatives to constrnatives to constrnatives to constrnatives to construct a coheruct a coheruct a coheruct a coheruct a coherentententententparagraph.paragraph.paragraph.paragraph.paragraph.

1.1.1.1.1. A. In rejecting the functionalismin positivist organisationtheory, either wholly orpartially, there is often a movetowards a political model oforganisation theory.

B. Thus the analysis would shiftto the power resourcespossessed by different groupsin the organisation and theway they use these resourcesin actual power plays to shapethe organisational structure.

C. At the extreme, in one set ofwritings, the growth ofadministrators in theorganisation is held to becompletely unrelated to thework to be done and to becaused totally by the politicalpursuit of self-interest.

D. The political model holds thatindividual interests are pursued

Section Iin organisational life throughthe exercise of power andinfluence.

(1) ADBC (2) CBAD(3) DBCA (4) ABDC

2.2.2.2.2. A. Group decision making,however, does not necessarilyfully guard against arbitrari-ness and anarchy, forindividual capriciousness canget substituted by collusion ofgroup members.

B. Nature itself is an intricatesystem of checks and balances,meant to preserve the delicatebalance between variousenvironmental factors thataffect our ecology.

C. In institutions also, there isa need to have in place asystem of checks and balanceswhich inhibits the concentrationof power in only someindividuals.

D. When human interventionsalter this delicate balance, theoutcomes have been seen tobe disastrous.

(1) CDAB (2) BCAD(3) CABD (4) BDCA

Sample Test Questions from IIMs CATBulletin for December 2000 Exam

For the Session beginning July 2001

Page 2: Cat 1

2 MBA ENTRANCE EXAMINATION AT A GLANCE

3.3.3.3.3. A. He was bone-weary and soul-weary, and found himselfmuttering, “Either I can’t managethis place, or it’s unmanage-able.”

B. To his horror, he realised thathe had become the victim ofan amorphous, unwitting,unconscious conspiracy toimmerse him in routine workthat had no significance.

C. It was one of those nights inthe office when the office clockwas moving towards four inthe morning and Bennis wasstill not through with theincredible mass of paperstacked before him.

D. He reached for his calendarand ran his eyes down eachhour, half-hour, and quarter-hour, to see where his time hadgone that day, the day before,the month before.

(1) ABCD (2) CADB(3) BDCA (4) DCBA

4.4.4.4.4. A. With that, I swallowed theshampoo, and obtained mostrealistic results almost on thespot.

B. The man shuffled away into theback regions to make up aprescription, and after a momentI got through on the shop-telephone to the Consulate,intimating my location.

C. Then, while the pharmacist waswrapping up a six-ounce bottleof the mixture, I groaned andinquired whether he could giveme something for acute gastriccramp.

D. I intended to stage a sharpgastric attack, and entering anold-fashioned pharmacy,

I asked for a popular shampoomixture, consisting of olive oiland flaked soap.

(1) DCBA (2) DACB(3) BDAC (4) BCDA

5.5.5.5.5. A. Since then, intelligence testshave been mostly used toseparate dull children in schoolfrom average or bright children,so that special education canbe provided to the dull.

B. In other words, intelligence testsgive us a norm for each age.

C. Intelligence is expressed asintelligence quotient, and testsare developed to indicate whatan average child of a certainage can do—what a 5-year-old can answer, but a 4-year-old cannot, for instance.

D. Binet developed the first set ofsuch tests in the early 1900sto find out which children inschool needed specialattention.

E. Intelligence can be measuredby tests.

(1) CDABE (2) DECAB(3) EDACB (4) CBADE

Directions (Qs. 6 to 13) : Directions (Qs. 6 to 13) : Directions (Qs. 6 to 13) : Directions (Qs. 6 to 13) : Directions (Qs. 6 to 13) : Read eachRead eachRead eachRead eachRead eachof the eight shorof the eight shorof the eight shorof the eight shorof the eight short passages given belowt passages given belowt passages given belowt passages given belowt passages given belowand answer these questions followingand answer these questions followingand answer these questions followingand answer these questions followingand answer these questions followingthe passages :the passages :the passages :the passages :the passages :

6.6.6.6.6. Three airlines—IA, JA and SA—operate on the Delhi-Mumbai route. Toincrease the number of seats sold, SAreduced its fares and this was emulatedby IA and JA immediately. The generalbelief was that the volume of air travelbetween Delhi and Mumbai wouldincrease as a result.

Which of the following, if true, wouldadd credence to the general belief ?

(1) Increase in profitability of the threeairlines.

Page 3: Cat 1

MBA ENTRANCE EXAMINATION AT A GLANCE 3

(2) Extension of the discount schemeto other routes.

(3) A study that shows that air travellersin India are price-conscious.

(4) A study that shows that as muchas 80% of air travel in India iscompany-sponsored.

7. 7. 7. 7. 7. According to McNeill, a Brahminpriest was expected to be able to reciteat least one of the Vedas. The practicewas essential for several centuries whenthe Vedas had not yet been writtendown. It must have had a selective effect,since priests would have been recruitedfrom those able or willing to memoriselong passages. It must have helped inthe dissemination of the work, since amemorised passage can be duplicatedmany times.

Which one of the following can beinferred from the above passage ?

(1) Reciting the Vedas was aBrahmin’s obligation.

(2) The Vedic priest was like arecorded audio cassette.

(3) McNeill studied the behaviour ofBrahmin priests.

(4) Vedic hymns had not beenscripted.

8. 8. 8. 8. 8. Developed countries have madeadequate provisions for social security forsenior citizens. State insurers (as well asprivate ones) offer medicare and pensionbenefits to people who can no longerearn. In India, with the collapse of thejoint family system, the traditional shelterof the elderly has disappeared. And aState faced with a financial crunch is notin a position to provide social security.So, it is advisable that the workingpopulation give serious thought to buildinga financial base for itself.

Which one of the following, if it wereto happen, weakens the conclusiondrawn in the above passage the most ?

(1) The investible income of the working

population, as a proportion of itstotal income, will grow in the future.

(2) The insurance sector is under-developed and trends indicate thatit will be extensively privatised inthe future.

(3) India is on a path of developmentthat will take it to a developedcountry status, with all its positiveand negative implications.

(4) If the working population builds astronger financial base, there willbe a revival of the joint family system.

9. 9. 9. 9. 9. Various studies have shown thatour forested and hilly regions and, ingeneral, areas where biodiversity—asreflected in the variety of flora—is high,are the places where poverty appearsto be high. And these same areas arealso the ones where educationalper formance seems to be poor.Therefore, it may be surmised that, evendisregarding poverty status, richness inbiodiversity goes hand in hand witheducational backwardness.

Which one of the followingstatements, if true, can be said to bestprovide supporting evidence for thesurmise mentioned in the passage ?

(1) In regions where there is little varietyin flora, educational performanceis seen to be as good as in regionswith high variety in flora, whenpoverty levels are high.

(2) Regions which show highbiodiversity also exhibit pooreducational performance, at lowlevels of poverty.

(3) Regions which show high bio-diversity reveal high levels of povertyand poor educational performance.

(4) In regions where there is lowbiodiversity, at all levels of poverty,educational performance is seento be good.

Page 4: Cat 1

4 MBA ENTRANCE EXAMINATION AT A GLANCE

10.10.10.10.10. Cigarettes constitute a mere 20%of tobacco consumption in India, andfewer than 15% of the 200 milliontobacco users consume cigarettes. Yetthese 15% contribute nearly 90% of thetax revenues to the Exchequer from thetobacco sector. The punitive cigarettetaxation regime has kept the tax basenarrow, and reducing taxes will expandthis base.

Which one of the following bestbolsters the conclusion that reducingduties will expand the tax base ?

(1) The cigarette manufacturers’association has decided to indulgein aggressive promotion.

(2) There is a likelihood that tobaccoconsumers will shift to cigarettesmoking if cigarette prices wereto reduce.

(3) The cigarette manufacturers arelobbying for a reduction on duties.

(4) An increase in duties on non-cigarette tobacco may leadto a shift in favour of cigarettesmoking.

11.11.11.11.11. Thomas Malthus, the Britishclergyman turned economist, predictedthat the planet would not be able tosupport the human population for long.His explanation was that humanpopulation grows at a geometric rate,while the food supply grows only at anarithmetic rate.

Which one of the following, if true,would not undermine the thesis offeredby Malthus ?

(1) Population growth can be sloweddown by the voluntary choices ofindividuals and not just by naturaldisasters.

(2) The capacity of the planet to feeda growing human population canbe enhanced through bio-technological means.

(3) Human systems, and naturalsystems like food supply, follownatural laws of growth which haveremained constant, and willremain unchanged.

(4) Human beings can colonise otherplanetary systems on a regular andon-going basis to accommodate agrowing population.

12. 12. 12. 12. 12. The company’s coffee crop for1998-99 totalled 8079 tonnes, an alltime record. The increase over theprevious year’s production of 5830tonnes was 38.58%. The previoushighest crop was 6089 tonnes in 1970-71. The company had fixed a target of8000 tonnes to be realised by the year2000-01 and this has been achievedtwo years earlier, thanks to the emphasislaid on the key areas of irrigation,replacement of unproductive coffeebushes, intensive refilling and improvedagricultural practices. It is now ourendeavour to reach the target of 10000tonnes in the year 2001-02.

Which one of the followingwould contribute most to making thetarget of 10000 tonnes in 2001-02unrealistic ?

(1) The potential of the productivityenhancing measures implementedup to now has been exhausted.

(2) The total company land undercoffee has remained constant since1969 when an estate in the NilgiriHills was acquired.

(3) The sensitivity of the crop toclimatic factors makes predictionsabout production uncertain.

(4) The target-setting proceduresin the company have beenproved to be sound by theachievement of the 8000 tonnetarget.

Page 5: Cat 1

MBA ENTRANCE EXAMINATION AT A GLANCE 5

13. 13. 13. 13. 13. Animals in general are shrewd inproportion as they cultivate society.Elephants and beavers show thegreatest signs of this sagacity whenthey are together in large numbers, butwhen man invades their communitiesthey lose all their spirit of industry.Among insects, the labours of the beeand the ant have attracted the attentionand admiration of naturalists, but all theirsagacity seems to be lost uponseparation, and a single bee or antseems destitute of every degree ofindustry. It becomes the most stupidinsect imaginable, and it languishes andsoon dies.

Which of the following can beinferred from the above passage ?

(1) Humankind is responsible for thedestruction of the natural habitatof animals and insects.

(2) Animals, in general, are unableto function effectively outside theirnormal social environment.

(3) Naturalists have great admirationfor bees and ants, despite theirlack of industry upon separation.

(4) Elephants and beavers are smarterthan bees and ants in the presenceof human beings.

Directions (Qs. 14 & 15) : Directions (Qs. 14 & 15) : Directions (Qs. 14 & 15) : Directions (Qs. 14 & 15) : Directions (Qs. 14 & 15) : For eachFor eachFor eachFor eachFor eachof these questions, indicate which ofof these questions, indicate which ofof these questions, indicate which ofof these questions, indicate which ofof these questions, indicate which ofthe statements given withe statements given withe statements given withe statements given withe statements given with that parth that parth that parth that parth that particularticularticularticularticularquestion is consistent with the descriptionquestion is consistent with the descriptionquestion is consistent with the descriptionquestion is consistent with the descriptionquestion is consistent with the descriptionof the unseasonable man in the passageof the unseasonable man in the passageof the unseasonable man in the passageof the unseasonable man in the passageof the unseasonable man in the passagegiven below :given below :given below :given below :given below :

Unseasonableness is a tendency todo socially permissible things at thewrong time. The unseasonable man isthe sort of person who comes to confidein you when you are busy. He serenadeshis beloved when she is ill. He asks aman who has just lost money by paying

a bill for a friend to pay a bill for him.He invites a friend to go for a ride justafter the friend has finished a long cartrip. He is eager to offer services whichare not wanted but which cannot bepolitely refused. If he is present at anarbitration, he stirs up dissensionbetween the two parties, who werereally anxious to agree. Such is theunseasonable man.

14.14.14.14.14. He tends to(1) Entertain women.(2) Be a successful arbitrator

when dissenting parties areanxious to agree.

(3) Be helpful when solicited.(4) Tell a long story to people who

have heard it many timesbefore.

15.15.15.15.15. The unseasonable man tends to(1) Bring a higher bidder to a

salesman who has just closeda deal.

(2) Disclose confidential informa-tion to others.

(3) Sing the praises of the bridewhen he goes to a wedding.

(4) Sleep late and rise early.Directions (Qs. 16 to 23) : Directions (Qs. 16 to 23) : Directions (Qs. 16 to 23) : Directions (Qs. 16 to 23) : Directions (Qs. 16 to 23) : In each ofIn each ofIn each ofIn each ofIn each of

the following sentences, a parthe following sentences, a parthe following sentences, a parthe following sentences, a parthe following sentences, a part of thet of thet of thet of thet of thesentence is underlined. Beneath eachsentence is underlined. Beneath eachsentence is underlined. Beneath eachsentence is underlined. Beneath eachsentence is underlined. Beneath eachsentence, four different ways of phrasingsentence, four different ways of phrasingsentence, four different ways of phrasingsentence, four different ways of phrasingsentence, four different ways of phrasingthe underlined parthe underlined parthe underlined parthe underlined parthe underlined part art art art art are indicated.e indicated.e indicated.e indicated.e indicated.Choose the best alterChoose the best alterChoose the best alterChoose the best alterChoose the best alternative frnative frnative frnative frnative from amongom amongom amongom amongom amongthe given alterthe given alterthe given alterthe given alterthe given alternatives :natives :natives :natives :natives :

16. 16. 16. 16. 16. It was us who had left before hearrived.

(1) We who had left before time hehad arrived.

(2) Us who had went before hearrived.

(3) Us who had went before hadarrived.

(4) We who had left before hearrived.

Page 6: Cat 1

6 MBA ENTRANCE EXAMINATION AT A GLANCE

17. 17. 17. 17. 17. The MP rose up to say that, inher opinion, she thought the Women’sReservation Bill should be passed onunanimously.

(1) Rose to say that she thought theWomen’s Reservation Bill shouldbe passed

(2) Rose up to say that, the Women’sReservation Bill should be passedon

(3) Rose to say that, in her opinion,she thought that the Women’sReservation Bill should be passed

(4) Rose to say that, in her opinion,the Women’s Reservation Billshould be passed on

18. 18. 18. 18. 18. Mr. Pillai, the president of theunion and who is also a member of thecommunity group, will be in charge ofthe negotiations.

(1) Since he is a member of thecommunity group

(2) Also being a member of thecommunity group

(3) A member of the community group(4) , in addition, who is a member of

the community group19. 19. 19. 19. 19. Since the advent of cable

television, at the beginning of thisdecade, the entertainment industry tooka giant stride forward in our country.

(1) This decade saw the entertainmentindustry taking

(2) This decade, the entertainmentindustry has taken

(3) This decade, the entertainmentindustry had taken

(4) This decade, the entertainmentindustry took

20. 20. 20. 20. 20. His mother made great sacrificesto educate him, moving house on threeoccasions, and severing the thread onher loom’s shuttle whenever Mencius

neglected his lessons to make himunderstand the need to persevere.

(1) Severing the thread on her loom’sshuttle whenever Menciusneglected his lessons to make himunderstand the need to persevere.

(2) Severed the thread on her loom’sshuttle whenever Menciusneglected his lessons to make himunderstand the need to persevere.

(3) Severed the thread on her loom’sshuttle whenever Mencius neglectedhis lessons to make him understandthe need for persevering.

(4) Severing the thread on her loom’sshuttle whenever Menciusneglected his lessons, to makethem understand the need topersevere.

21. 21. 21. 21. 21. If you are on a three-monthsoftware design project and, in twoweeks, you’ve put together a programmethat solves part of the problem, show itto your boss without delay.

(1) And, you’ve put together aprogramme that solves part of theproblem in two weeks

(2) And, in two weeks, you’ve puttogether a programme that solvespart of the problem

(3) And, you’ve put together aprogramme that has solved partof the problem in two weeks

(4) And, in two weeks you puttogether a programme that solvedonly part of the problem

22. 22. 22. 22. 22. Many of these environmentalistsproclaim to save nothing less than theplanet itself.

(1) To save nothing lesser than(2) That they are saving nothing lesser

than(3) To save nothing less than(4) That they save nothing less than

Page 7: Cat 1

MBA ENTRANCE EXAMINATION AT A GLANCE 7

23. 23. 23. 23. 23. Bacon believes that the medicalprofession should be permitted to easeand quicken death where the end wouldotherwise only delay for a few daysand at the cost of great pain.

(1) Be delayed for a few days(2) Be delayed for a few days and(3) Be otherwise only delayed for a

few days and(4) Otherwise only delay for a few

days andDirections (Qs. 24 to 50) : Directions (Qs. 24 to 50) : Directions (Qs. 24 to 50) : Directions (Qs. 24 to 50) : Directions (Qs. 24 to 50) : Each ofEach ofEach ofEach ofEach of

the five passages given below isthe five passages given below isthe five passages given below isthe five passages given below isthe five passages given below isfollowed by questions. Read each offollowed by questions. Read each offollowed by questions. Read each offollowed by questions. Read each offollowed by questions. Read each ofthese passages and then answer thethese passages and then answer thethese passages and then answer thethese passages and then answer thethese passages and then answer thequestions following each of thesequestions following each of thesequestions following each of thesequestions following each of thesequestions following each of thesepassages :passages :passages :passages :passages :

PASSAGE IPASSAGE IPASSAGE IPASSAGE IPASSAGE IThe World Trade Organisation (WTO)

was created in the early 1990s as acomponent of the Uruguay Roundnegotiation. However, it could havebeen negotiated as part of the TokyoRound of the 1970s, since thatnegotiation was an attempt as a‘constitutional reform’ of the GeneralAgreement on Tariffs and Trade (GATT).Or it could have been put off to thefuture, as the US government wanted.What factors led to the creation of theWTO in the early 1990s ?

One factor was the pattern ofmultilateral bargaining that developedlate in the Uruguay Round. Like allcomplex international agreements, theWTO was a product of a series of trade-offs between principal actors andgroups. For the United States, which didnot want a new organisation, the disputesettlement part of the WTO packageachieved its long-standing goal of amore effective and more legal disputesettlement system. For the Europeans,who by the 1990s had come to viewGATT dispute settlement less in political

terms and more as a regime of legalobligations, the WTO package wasacceptable as a means to discipline theresort to unilateral measures by theUnited States. Countries like Canadaand other middle and smaller tradingpartners were attracted by the expansionof a rules-based system and by thesymbolic value of a trade organisation,both of which inherently support theweak against the strong. The developingcountries were attracted due to theprovisions banning unilateral measures.Finally, and perhaps most important,many countries at the Uruguay Roundcame to put a higher priority on theexport gains than on the import lossesthat the negotiation would produce, andthey came to associate the WTO and arules-based system with those gains. Thisreasoning—replicated in many countries—was contained in U.S. AmbassadorKantor’s defence of the WTO, and itamounted to a recognition thatinternational trade and its benefitscannot be enjoyed unless trading nationsaccept the discipline of a negotiatedrules-based environment.

A second factor in the creation of theWTO was pressure from lawyers andthe legal process. The dispute settlementsystem of the WTO was seen as avictory of legalists over pragmatists butthe matter went deeper than that. TheGATT, and the WTO, are contractorganisations based on rules, and it isinevitable that an organisation createdto further rules will in turn be influencedby the legal process. Robert Hudec haswritten of the ‘momentum of legaldevelopment’, but what is this precisely?Legal development can be defined aspromotion of the technical legal valuesof consistency, clarity (or, certainty) and

Page 8: Cat 1

8 MBA ENTRANCE EXAMINATION AT A GLANCE

effectiveness; these are values that thoseresponsible for administering any legalsystem will seek to maximise. As itplayed out in the WTO, consistencymeant integrating under one roof thewhole lot of separate agreements signedunder GATT auspices; clarity meantremoving ambiguities about the powersof contracting parties to make certaindecisions or to undertake waivers; andeffectiveness meant eliminatingexceptions arising out of grandfather-rights and resolving defects in disputesettlement procedures and institutionalprovisions. Concern for these values isinherent in any rules-based system ofco-operation, since without these valuesrules would be meaningless in the firstplace. Rules, therefore, create their ownincentive for fulfillment.

The momentum of legal developmenthas occurred in other institutions besidesthe GATT, most notably in the EuropeanUnion (EU). Over the past two decadesthe European Court of Justice (ECJ) hasconsistently rendered decisions that haveexpanded incrementally the EU’s internalmarket, in which the doctrine of ‘mutualrecognition’ handed down in the caseCassis de Dijon in 1979 was a keyturning point. The Court is now widelyrecognised as a major player inEuropean integration, even thougharguably such a strong role was notoriginally envisaged in the Treaty ofRome, which initiated the currentEuropean Union. One means the Courtused to expand integration was the‘teleological method of interpretation’,whereby the actions of member stateswere evaluated against ‘theaccomplishment of the most elementarycommunity goals set forth in thePreamble to the [Rome] treaty’. The

teleological method represents an effortto keep current policies consistent withstated goals, and it is analogous to theeffort in GATT to keep contracting partytrade practices consistent with statedrules. In both cases legal concerns andprocedures are an independent forcefor further cooperation.

In large part the WTO was anexercise in consolidation. In the contextof a trade negotiation that created anear-revolutionary expansion ofinternational trade rules, the formationof the WTO was a deeply conservativeact needed to ensure that the benefitsof the new rules would not be lost. TheWTO was all about institutional structureand dispute settlement: these are theconcerns of conservatives and notrevolutionaries, which is why lawyersand legalists took the lead on theseissues. The WTO codified the GATTinstitutional practice that had developedby custom over three decades, and itincorporated a new dispute settlementsystem that was necessary to keep bothold and new rules from becoming asham. Both the international structure andthe dispute settlement system werenecessary to preserve and enhance theintegrity of the multilateral trade regimethat had been built incrementally fromthe 1940s to the 1990s.

24. 24. 24. 24. 24. What could be the closest reasonwhy the WTO was not formed in the1970s ?

(1) The US government did not like it.(2) Important players did not find it in

their best interest to do so.(3) Lawyers did not work for the

dispute settlement system.(4) The Tokyo Round negotiation

was an attempt at constitutionalreform.

Page 9: Cat 1

MBA ENTRANCE EXAMINATION AT A GLANCE 9

25. 25. 25. 25. 25. The most likely reason for theacceptance of the WTO package bynations was that

(1) It had the means to prevent the USfrom taking unilateral measures.

(2) They recognised the need for arule-based environment to protectthe benefits of increased trade.

(3) It settles disputes more legally andmore effectively.

(4) Its rule-based system leads toexport gains.

26. 26. 26. 26. 26. According to the passage, WTOpromoted the technical legal valuespartly through

(1) Integrating under one roof theagreements signed under GATT.

(2) Rules that create their ownincentive for fulfillment.

(3) Grandfather-rights exceptions anddefects in dispute settlementprocedures.

(4) Ambiguities about the powers ofcontracting parties to make certaindecisions.

27. 27. 27. 27. 27. In the method of interpretation ofthe European Court of Justice,

(1) Current policies needed to beconsistent with stated goals.

(2) Contracting party trade practicesneeded to be consistent with statedrules.

(3) Enunciation of the most elementarycommunity goals needed to beemphasised.

(4) Actions of member states neededto be evaluated against the statedcommunity goals.

28. 28. 28. 28. 28. In the settlement “...it amountedto a recognition that international tradeand its benefits cannot be enjoyed unlesstrading nations accept the discipline ofa negotiated rules-based environment”,‘it’ refers to :

(1) Ambassador Kantor’s defence ofthe WTO.

(2) The higher priority on export gainsplaced by many countries at theUruguay Round.

(3) The export gains many countriescame to associate with a rule-based system.

(4) The provision of a rule-basedsystem by the WTO.

29. 29. 29. 29. 29. The importance of Cassis de Dijonis that it

(1) Gave a new impetus to themomentum of legal developmentat the European Court of Justice.

(2) Resulted in a decision thatexpanded incrementally the EU’sinternal market.

(3) Strengthened the role of the Courtmore than envisaged in the Treatyof Rome.

(4) Led to a doctrine that was a key turn-ing point in European integration.

PASSAGE IIPASSAGE IIPASSAGE IIPASSAGE IIPASSAGE II

Have you ever come across apainting, by Picasso, Mondrian, Miro,or any modern abstract painter of thiscentury, and found yourself engulfed ina brightly coloured canvas which yoursenses cannot interpret ? Many peoplewould tend to denounce abstractionismas senseless trash. These people aredisoriented by Miro’s bright, fancifulcreatures and two-dimensional canvases.They click their tongues and shake theirheads at Mondrian’s grid works,declaring the poor guy played too manyscrabble games. They silently shake theirheads in sympathy for Picasso, whosegruesome, distorted figures must be areflection of his mental health. Then,standing in front of a work by CharlieRussell, the famous Western artist, they’ll

Page 10: Cat 1

10 MBA ENTRANCE EXAMINATION AT A GLANCE

declare it a work of God. People feelmore comfortable with something theycan relate to and understandimmediately without too much thought.This is the case with the work of CharlieRussell. Being able to recognise theelements in his paintings—trees, horsesand cowboys—gives people a safety lineto their world of “reality”. There aresome who would disagree when I sayabstract art requires more creativity andartistic talent to produce a good piecethan does representational art, but thereare many weaknesses in their arguments.

People who look down on abstractart have several major arguments tosupport their beliefs. They feel that artiststurn abstract because they are notcapable of the technical drafting skillsthat appear in a Russell; therefore, suchartists create an art form that anyone iscapable of and that is less timeconsuming, and then parade it as artisticprogress. Secondly, they feel thatpurpose of art is to create something ofbeauty in an orderly, logicalcomposition. Russell’s compositions arebalanced and rational; everything sitscalmly on the canvas, leaving the viewersatisfied that he has seen all there is tosee. The modern abstractionists, on theother hand, seem to compose theirpieces irrationally. For example, uponseeing Picasso’s Guernica, a friend ofmine asked me, “What’s the point?”Finally, many people feel that art shouldportray the ideal and real. The exactnessof detail in Charlie Russell’s work is anexample of this. He has been called agreat historian because his pieces depictthe life style, dress, and events of thetimes. His subject matter is derived fromhis own experiences on the trail, andreproduced to the smallest detail.

I agree in part with many of thesearguments, and at one time even endorsed

them. But now, I believe differently. Firstly,I object to the argument that abstractartists are not capable of drafting. Manyabstract artists, such as Picasso, areexcellent draftsmen. As his work matured,Picasso became more abstract in orderto increase the expressive quality of hiswork. Guernica was meant as a protestagainst the bombing of that city by theGermans. To express the terror andsuffering of the victims more vividly, hedistorted the figures and presented themin a black and white journalistic manner.If he had used representational imagesand colour, much of the emotional contentwould have been lost and the piecewould not have caused the demand forjustice that it did. Secondly, I do not thinkthat a piece must be logical andaesthetically pleasing to be art. Themessage it conveys to its viewers is moreimportant. It should reflect the ideals andissues of its time and be true to itself, notjust a flowery, glossy surface. For example,through his work, Mondrian was tryingto present a system of simplicity, logic,and rational order. As a result, his piecesdid end up looking like a scrabble board.Miro created powerful, surrealistic imagesfrom his dreams and subconscious. Theseartists were trying to evoke a responsefrom society through an expressionisticmanner. Finally, abstract artistsand representational artists maintaindifferent ideas about ‘reality’. To therepresentational artist, reality is what hesees with his eyes. This is the reality hereproduces on canvas. To the abstractartist, reality is what he feels about whathis eyes see. This is the reality he interpretson canvas. This can be illustrated byMondrian’s Trees series. You can actuallysee the progression from the earlyrecognisable, though abstracted, Trees,to his final solution, the grid system.

Page 11: Cat 1

MBA ENTRANCE EXAMINATION AT A GLANCE 11

A cycle of abstract and representa-tional art began with the first scratchingof prehistoric man. From the abstractionsof ancient Egypt to representational,classical Rome, returning toabstractionism in early Christian art andso on up to the present day, the cyclehas been going on. But this day andage may witness its death through thecamera. With film, there is no need toproduce finely detailed, historicalrecords manually; the camera does thisfor us more efficiently. Maybe,representational art would cease to exist.With abstractionism as the victor of thefirst battle, may be a different kind ofcycle will be touched off. Possibly, sometime in the distant future, thousands ofyears from now, art itself will bephysically non-existent. Some artiststoday believe that once they haveplanned and constructed a piece in theirmind, there is no sense in finishing itwith their hands; it has already beendone and can never be duplicated.

30. 30. 30. 30. 30. The author argues that manypeople look down upon abstract artbecause they feel that

(1) Modern abstract art does notportray what is ideal and real.

(2) Abstract artists are unskilled inmatters of technical drafting.

(3) Abstractionists compose irra-tionally.

(4) All of the above.31.31.31.31.31. The author believes that people

feel comfortable with representational artbecause

(1) They are not engulfed in brightlycoloured canvases.

(2) They do not have to click theirtongues and shake their heads insympathy.

(3) They understand the art withoutputting too much strain on theirminds.

(4) Paintings like Guernica do nothave a point.

32.32.32.32.32. In the author’s opinion, Picasso’sGuernica created a strong demand forjustice since

(1) It was a protest against theGerman bombing of Guernica.

(2) Picasso managed to express theemotional content well with hisabstract depiction.

(3) It depicts the terror and sufferingof the victims in a distortedmanner.

(4) It was a mature work of Picasso’s,painted when the artist’s draftingskills were excellent.

33.33.33.33.33. The author acknowledgesthat Mondrian’s pieces may have endedup looking like a scrabble boardbecause

(1) Many people declared the poorguy played too many scrabblegames.

(2) Mondrian believed in the ‘grid-works’ approach to abstractionistpainting.

(3) Mondrian was trying to conveythe message of simplicity andrational order.

(4) Mondrian learned from his Treesseries to evolve a grid system.

34.34.34.34.34. The main difference between theabstract artist and the representationalartist in matters of the ‘ideal’ and the‘real’, according to the author, is

(1) How each chooses to deal with‘reality’ on his or her canvas.

(2) The superiority of interpretationof reality over reproduction ofreality.

(3) The different values attached byeach to being a historian.

(4) The varying levels of drafting skillsand logical thinking abilities.

Page 12: Cat 1

12 MBA ENTRANCE EXAMINATION AT A GLANCE

PASSAGE IIIPASSAGE IIIPASSAGE IIIPASSAGE IIIPASSAGE III

Each one has his reasons : for oneart is a flight; for another, a means ofconquering. But one can flee into ahermitage, into madness, into death.One can conquer by arms. Why doesit have to be writing, why does onehave to manage his escapes andconquests by writing ? Because, behindthe various aims of authors, there is adeeper and more immediate choicewhich is common to all of us. We shalltry to elucidate his choice, and we shallsee whether it is not in the name of thisvery choice of writing that theengagement of writers must be required.

Each of our perceptions isaccompanied by the consciousness thathuman reality is a ‘revealer’, that is, itis through human reality that ‘there is’being, or, to put it differently, that manis the means by which things aremanifested. It is our presence in theworld which multiplies relations. It is wewho set up a relationship between thistree and that bit of sky. Thanks to us,that star which has been dead formillennia, that quarter moon, and thatdark river are disclosed in the unity ofa landscape. It is the speed of our autoand our airplane which organises thegreat masses of the earth. With each ofour acts, the world reveals to us a newface. But, if we know that we aredirectors of being, we also know thatwe are not its producers. If we turn awayfrom this landscape, it will sink backinto its dark permanence. At least, itwill sink back; there is no one madenough to think that it is going to beannihilated. It is we who shall beannihilated, and the earth will remainin its lethargy until another consciousnesscomes along to awaken it. Thus, to our

inner certainty of being ‘revellers’ isadded that of being inessential inrelation to the thing revealed.

One of the chief motives of artisticcreation is certainly the need of feelingthat we are essential in relationship tothe world. If I fix on canvas or in writinga certain aspect of the fields or sea ora look on someone’s face which I havedisclosed, I am conscious of havingproduced them by condensingrelationships, by introducing order wherethere was none, by imposing the unityof mind on the diversity of things. Thatis, I think myself essential in relation tomy creation. But this time it is the createdobject which escapes me; I can notreveal and produce at the same time.The creation becomes inessential inrelation to the creative activity. First ofall, even if it appears to others asdefinitive, the created object alwaysseems to us in a state of suspension; wecan always change this line, that shade,that word. Thus, it never forces itself. Anovice painter asked his teacher, ‘Whenshould I consider my painting finished ?’And the teacher answered, ‘When youcan look at it in amazement and say toyourself “I’m the one who did that!”’

Which amounts to saying ‘never’. Forit is virtually considering one’s work withsomeone else’s eyes and revealing whathas been created. But it is self-evident thatwe are proportionally less conscious ofthe thing produced and more consciousof our productive activity. When it is amatter of poetry or carpentry, we workaccording to traditional norms, with toolswhose usage is codified; it is Heidegger’sfamous ‘they’ who are working with ourhands. In this case, the result can seemto us sufficiently strange to preserve itsobjectivity in our eyes. But if we ourselves

Page 13: Cat 1

MBA ENTRANCE EXAMINATION AT A GLANCE 13

produce the rules of production, themeasures, the criteria, and if our creativedrive comes from the very depths of ourheart, then we never find anything butourselves in our work. It is we who haveinvented the laws by which we judge it.It is our history, our love, our gaiety thatwe recognise in it. Even if we shouldregard it without touching it any further,we never receive from it that gaiety orlove. We put them into it. The resultswhich we have obtained on canvas orpaper never seem to us objective. Weare too familiar with the processes ofwhich they are the effects. These processesremain a subjective discovery; they areourselves, our inspiration, our ruse, andwhen we seek to perceive our work, wecreate it again, we repeat mentally theoperations which produced it; each ofits aspects appears as a result. Thus, inthe perception, the object is given as theessential thing and the subject as theinessential. The latter seeks essentiality inthe creation and obtains it, but then itis the object which becomes theinessential.

The dialectic is nowhere moreapparent than in the art of writing, forthe literary object is a peculiar top whichexists only in movement. To make it comeinto view a concrete act called readingis necessary, and it lasts only as longas this act can last. Beyond that, thereare only black marks on paper. Now,the writer can not read what he writes,whereas the shoemaker can put on theshoes he has just made if they are tohis size, and the architect can live inthe house he has built. In reading, oneforesees; one waits. He foresees the endof the sentence, the following sentence,the next page. He waits for them to

confirm or disappoint his foresights. Thereading is composed of a host ofhypotheses, followed by awakenings, ofhopes and deceptions. Readers arealways ahead of the sentence they arereading in a merely probable futurewhich partly collapses and partly comestogether in proportion as they progress,which withdraws from one page to thenext and forms the moving horizon ofthe literary object. Without waiting,without a future, without ignorance, thereis no objectivity.

35.35.35.35.35. The author holds that(1) There is an objective reality

and a subjective reality.(2) Nature is the sum total of

disparate elements.(3) It is human action that reveals

the various facets of nature.(4) Apparently disconnected

elements in nature are unifiedin a fundamental sense.

36.36.36.36.36. It is the author’s contention that(1) Artistic creations are results of

human consciousness.(2) The very act of artistic creation

leads to the escape of thecreated object.

(3) Man can produce and revealat the same time.

(4) An act of creation forces itselfon our consciousness leavingus full of amazement.

37.37.37.37.37. The passage makes a distinctionbetween perception and creation interms of

(1) Objectivity and subjectivity.(2) Revelation and action.(3) Objective reality and perceived

reality.(4) Essentiality and non-essentiality of

objects and subjects.

Page 14: Cat 1

14 MBA ENTRANCE EXAMINATION AT A GLANCE

38.38.38.38.38. The art of writing manifests thedialectic of perception and creationbecause

(1) Reading reveals the writing tillthe act of reading lasts.

(2) Writing to be meaningful needsthe concrete act of reading.

(3) This art is anticipated andprogresses on a series ofhypotheses.

(4) This literary object has amoving horizon brought aboutby the very act of creation.

39.39.39.39.39. A writer, as an artist,(1) Reveals the essentiality of

revelation.(2) Makes us feel essential vis-a-

vis nature.(3) Creates reality.(4) Reveals nature in its

permanence.

PASSAGE IVPASSAGE IVPASSAGE IVPASSAGE IVPASSAGE IV

Since World War II, the nation-statehas been regarded with approval byevery political system and everyideology. In the name of modernisationin the West, of socialism in the Easternbloc, and of development in the ThirdWorld, it was expected to guaranteethe happiness of individuals as citizensand of peoples as societies. However,the state today appears to have brokendown in many parts of the world. It hasfailed to guarantee either security orsocial justice, and has been unable toprevent either international wars or civilwars. Disturbed by the claims ofcommunities within it, the nation-statetries to repress their demands and toproclaim itself as the only guarantor ofsecurity or all. In the name of nationalunity, territorial integrity, equality of allits citizens and non-partisan secularism,the state can use its powerful resourcesto reject the demands of the

communities; it may even go sofar as genocide to ensure that orderprevails.

As one observes the awakening ofcommunities in different parts of theworld, one cannot ignore the context inwhich identity issues arise. It is no longera context of sealed frontiers andisolated regions but is one of integratedglobal systems. In a reaction to thistrend towards globalisation, individualsand communities everywhere arevoicing their desire to exist, to usetheir power of creation and to play anactive part in national and internationallife.

There are two ways in which thecurrent upsurge in demands for therecognition of identities can be lookedat. On the positive side, the efforts bycertain population groups to assert theiridentity can be regarded as “liberationmovements”, challenging oppressionand injustice. What these groups aredoing – proclaiming that they aredifferent, rediscovering the roots oftheir culture or strengthening groupsolidarity—may accordingly be seen aslegitimate attempts to escape from theirstate of subjugation and enjoy a certainmeasure of dignity. On the downside,however, militant action for recognitiontends to make such groups more deeplyentrenched in their attitude and to maketheir cultural compartments even morewatertight. The assertion of identity thenstarts turning into self-absorption andisolation, and is liable to slide intointolerance of others and towards ideasof “ethnic cleansing”, xenophobia andviolence.

Whereas continuous variations amongpeoples prevent drawing of cleardividing lines between the groups, those

Page 15: Cat 1

MBA ENTRANCE EXAMINATION AT A GLANCE 15

militating for recognition of their group’sidentity arbitrarily choose a limitednumber of criteria such as religion,language, skin colour, and place oforigin so that their members recognisethemselves primarily in terms of thelabels attached to the group whoseexistence is being asserted. Thisdistinction between the group in questionand other groups is established bysimplifying the feature selected.Simplification also works by transforminggroups into essences, abstractionsendowed with the capacity to remainunchanged through time. In some cases,people actually act as though the grouphas remained unchanged and talk, forexample, about the history of nationsand communities as if these entitiessurvived for centuries without changing,with the same ways of acting andthinking, the same desires, anxieties, andaspirations.

Paradoxically, precisely becauseidentity represents a simplifying fiction,creating uniform groups out of disparatepeople, that identity performs a cognitivefunction. It enables us to put names toourselves and others, form some ideaof who we are and who others are,and ascertain the place we occupyalong with the others in the world andsociety. The current upsurge to assertthe identity of groups can thus be partlyexplained by the cognitive functionperformed by identity. However, thatsaid, people would not go along asthey do, often in large numbers, withthe propositions put to them, in spite ofthe sacrifices they entail, if therewas not a very strong feeling ofneed for identity, a need to take stockof things and know “who we are”,

“where we come from” and “where weare going“.

Identity is thus a necessity in aconstantly changing world, but it canalso be a potent source of violence anddisruption. How can these twocontradictory aspects of identity bereconciled ? First, we must bear thearbitrary nature of identity categories inmind, not with a view to eliminating allforms of identification—which would beunrealistic since identity is a cognitivenecessity—but simply to remind ourselvesthat each of us has several identities atthe same time. Second, since tears ofnostalgia are being shed over the past,we recognise that culture is constantlybeing recreated by cobbling togetherfresh and original elements and counter-cultures. There are in our own countrya large number of syncretic cults whereinmodern elements are blended withtraditional values or people of differentcommunities venerate saints or divinitiesof particular faiths. Such cults andmovements are characterised by acontinual inflow and outflow of memberswhich prevent them from taking on aself-perpetuating existence of their ownand hold out hope for the future, indeed,perhaps for the only possible future.Finally, the nation-state must respond tothe identity urges of its constituentcommunities and to their legitimate questfor security and social justice. It must doso by inventing what the Frenchphilosopher and sociologist, RaymondAron, called “peace through law”. Thatwould guarantee justice both to the stateas a whole and its parts, and respectthe claims of both reason and emotions.The problem is one of reconcilingnationalist demands with the exerciseof democracy.

Page 16: Cat 1

16 MBA ENTRANCE EXAMINATION AT A GLANCE

40.40.40.40.40. According to the author,happiness of individuals was expectedto be guaranteed in the name of

(1) Development in the Third World.(2) Socialism in the Third World.(3) Development in the West.(4) Modernisation in the Eastern Bloc.41.41.41.41.41. Demands for recognition of

identities can be viewed(1) Positively and negatively.(2) As liberation movements and

militant action.(3) As efforts to rediscover cultural

roots which can slide towardsintolerance of others.

(4) All of the above.42.42.42.42.42. Going by the author’s exposition

of the nature of identity, which of thefollowing statements is untrue ?

(1) Identity represents creating uniformgroups out of disparate people.

(2) Identity is a necessity in thechanging world.

(3) Identity is a cognitive necessity.(4) None of the above43.43.43.43.43. According to the author, the

nation-state(1) Has fulfilled its potential.(2) Is willing to do anything to

preserve order.(3) Generates security for all its

citizens.(4) Has been a major force in prevent-

ing civil and international wars.44. 44. 44. 44. 44. Which of the following views of

the nation-state cannot be attributed tothe author ?

(1) It has not guaranteed peace andsecurity.

(2) It may go as far as genocide forself-preservation.

(3) It represents the demands ofcommunities within it.

(4) It is unable to prevent internationalwars.

PASSAGE VPASSAGE VPASSAGE VPASSAGE VPASSAGE VThe persistent patterns in the way

nations fight reflect their cultural andhistorical traditions and deeply rootedattitudes that collectively make up theirstrategic culture. These patterns provideinsights that go beyond what can belearnt just by comparing armaments anddivisions. In the Vietnam War, thestrategic tradition of the United Statescalled for forcing the enemy to fight amassed battle in an open area, wheresuperior American weapons wouldprevail. The United States was trying tore-fight World War II in the jungles ofSoutheast Asia, against an enemy withno intention of doing so.

Some British military historiansdescribe the Asian way of war as oneof indirect attacks, avoiding frontalattacks meant to overpower anopponent. This traces back to Asianhistory and geography: the greatdistances and harsh terrain have oftenmade it difficult to execute the sort ofopen field clashes allowed by the flatterrain and relatively compact size ofEurope. A very different strategictradition arose in Asia.

The bow and arrow were metaphorsfor an Eastern way of war. By its nature,the arrow is an indirect weapon. Firedfrom a distance of hundreds of yards, itdoes not necessitate immediate physicalcontact with the enemy. Thus, it can befired from hidden positions. When firedfrom behind a ridge, the barrage seemsto come out of nowhere, taking theenemy by surprise. The tradition of thiskind of fighting is captured in theclassical strategic writings of the East.The 2,000 years’ worth of Chinesewritings on war constitutes the mostsubtle writings on the subject in any

Page 17: Cat 1

MBA ENTRANCE EXAMINATION AT A GLANCE 17

language. Not until Clausewitz, did theWest produce a strategic theorist tomatch the sophistication of Sun-tzu,whose Art of War was written 2,300years earlier.

In Sun-tzu and other Chinese writings,the highest achievement of arms is todefeat an adversary without fighting. Hewrote : “To win one hundred victoriesin one hundred battles is not the acmeof skill. To subdue the enemy withoutfighting is the supreme excellence.”Actual combat is just one among manymeans towards the goal of subduing anadversary. War contains too manysurprises to be a first resort. It can leadto ruinous losses, as has been seen timeand again. It can have the unwantedeffect of inspiring heroic efforts in anenemy, as the United States learned inVietnam, and as the Japanese foundout after Pearl Harbor.

Aware of the uncertainties of a militarycampaign, Sun-tzu advocated war onlyafter the most thorough preparations.Even then it should be quick and clean.Ideally, the army is just an instrument todeal the final blow to an enemy alreadyweakened by isolation, poor morale,and disunity. Ever since Sun-tzu, theChinese have been seen as masters ofsubtlety who take measured actions tomanipulate an adversary without hisknowledge. The dividing line betweenwar and peace can be obscure. Lowlevel violence often is the backdrop toa larger strategic campaign. Theunwitting victim, focused on the day-to-day events, never realises what’shappening to him until it’s too late.History holds many examples. The VietCong lured French and U.S. infantrydeep into the jungle, weakening theirmorale over several years. The mobile

army of the United States was designedto fight on the plains of Europe, whereit could quickly move unhindered fromone spot to the next. The jungle didmore than make quick movementimpossible; broken down into smallerunits and scattered in isolated bases,US forces were deprived of the feelingof support and protection that ordinarilycomes from being part of a big army.

The isolation of U.S. troops in Vietnamwas not just a logistical detail, somethingthat could be overcome by, for instance,bringing in reinforcements by helicopter.In a big army reinforcements are readilyavailable. It was Napoleon who realisedthe extraordinary effects on morale thatcome from being part of a largerformation. Just the knowledge of it lowersthe soldier’s fear and increases hisaggressiveness. In the jungle and onisolated bases, this feeling was removed.The thick vegetation slowed down thereinforcements and made it difficult tofind stranded units. Soldiers felt theywere on their own.

More important, by altering the waythe war was fought, the Viet Congstripped the United States of its belief inthe inevitability of victory, as it had doneto the French before them. Morale washigh when these armies first went toVietnam. Only after many years ofdebilitating and demoralising fightingdid Hanoi launch its decisive attacks, atDienbienphu in 1954 and againstSaigon in 1975. It should be recalledthat in the final push to victory the NorthVietnamese abandoned their jungleguerrilla tactics completely, committingtheir entire army of twenty divisions topuhsing the South Vietnamese intocollapse. This final battle, with theenemy’s army all in one place, was the

Page 18: Cat 1

18 MBA ENTRANCE EXAMINATION AT A GLANCE

one that the United States haddesperately wanted to fight in 1965.When it did come out into the open in1975, Washington had alreadywithdrawn its forces and there was nopossibility of re-intervention.

The Japanese early in World War IIused a modern form of the indirectattack, one that relied on stealth andsurprise for its effect. At Pearl Harbor,in the Philippines, and in Southeast Asia,stealth and surprise were attained bysailing under radio silence so that thenavy’s movements could not be tracked.Moving troops aboard ships intoSouthest Asia made it appear that theJapanese army was also “invisible.”Attacks against Hawaii and Singapreseemed, to the American and Britishdefenders, to come from nowhere. InIndonesia and the Philippines theJapanese attack was even faster thanthe German blitz against France in theWest.

The greatest military surprises inAmerican history have all been in Asia.Surely there is something going on herebeyond the purely technical difficultiesof detecting enemy movements. PearlHarbor, the Chinese intervention inKorea, and the Tet offensive in Vietnamall came out of a tradition of surpriseand stealth. U.S. technical intelligence—the location of enemy units and theirmovements—was greatly improved aftereach surprise, but with no noticeableimprovement in the American ability toforesee or prepare what would happennext. There is a cultural divide here, notjust a technical one. Even when it waspossible to track an army withintelligence satellites, as when Iraqinvaded Kuwait or when Syria and Egyptattacked Israel, surprise was achieved.The United States was stunned by Iraq’s

attack on Kuwait even though it hadsatellite pictures of Iraqi troops massingat the border.

The exception that proves the pointthat cultural differences obscure theWest’s understanding of Asian behaviourwas the Soviet Union’s 1979 invasionof Afghanistan. This was fully anticipatedand understood in advance. There wasno surprise because the United Statesunderstood Moscow’s world view andthinking. It could anticipate Soviet actionalmost as well as the Soviets themselves,because the Soviet Union was really aWestern country.

The difference between the Easternand Western way of war is striking. TheWest’s great strategic writer, Clausewitz,linked war to politics, as did Sun-tzu.Both are opponents of militarism, ofturning war over to the generals. Butthere all similarity ends. Clausewitzwrote that the way to achieve a largerpolitical purpose is through destructionof the enemy’s army. After observingNapoleon conquer Europe by smashingenemy armies to bits, Clausewitz madehis famous remark in On War (1932)that combat is the continuation of politicsby violent means. Morale and unity areimportant, but they should be harnessedfor the ultimate battle. If the Eastern wayof war is embodied by the stealthyarcher, the metaphorical Westerncounterpart is the swordsman chargingforward, seeking a decisive showdown,eager to administer the blow that willobliterate the enemy once and for all.In this view, war proceeds along a fixedcourse and occupies a finite extent oftime, like a play in three acts with abeginning, a middle, and an end. Theend, the final scene, decides the issuesfor good.

When things don’t work out quite thisway, the Western military mind feels

Page 19: Cat 1

MBA ENTRANCE EXAMINATION AT A GLANCE 19

tremendous frustration. Sun-tzu’s greatdisciples, Mao Zedong and Ho ChiMinh, are respected in Asia fortheir clever use of indirection anddeception to achieve an advantage overstronger adversaries. But in the Westtheir approach is seen as underhandedand devious. To the American strategicmind, the Viet Cong guerrilla did notfight fairly. He should have come outinto the open and fought like a man,instead of hiding in the jungleand sneaking around like a cat in thenight.

45.45.45.45.45. According to the author, the mainreason for the U.S. losing the Vietnamwar was

(1) The Vietnamese understood thelocal terrain better.

(2) The lack of support for the warfrom the American people.

(3) The failure of the U.S. to mobiliseits military strength.

(4) Their inability to fight a war onterms other than those theyunderstood well.

46.46.46.46.46. Which of the following statementsdoes not describe the ‘Asian’ way ofwar ?

(1) Indirect attacks without frontalattacks.

(2) The swordsman charging forwardto obliterate the enemy once andfor all.

(3) Manipulation of an adversarywithout his knowledge.

(4) Subduing an enemy withoutfighting.

47.47.47.47.47. Which of the following is not oneof Sun-tzu’s ideas ?

(1) Actual combat is the principalmeans of subduing an adversary.

(2) War should be undertaken onlyafter thorough preparation.

(3) War is linked to politics.(4) War should not be left to the

generals alone.48.48.48.48.48. The difference in the concepts of

war of Clausewitz and Sun-tzu is bestcharacterised by

(1) Clausewitz’s support for militarismas against Sun-tzu’s opposition toit.

(2) Their relative degrees of sophisti-cation.

(3) Their attitude to guerrilla warfare.(4) Their differing conceptions of the

structure, time and sequences of awar.

49.49.49.49.49. To the Americans, the approachof the Viet Cong seemed deviousbecause

(1) The Viet Cong did not fight likemen out in the open.

(2) The Viet Cong allied withAmerica’s enemies.

(3) The Viet Cong took strategicadvice from Mao Zedong.

(4) The Viet Cong used bows andarrows rather than conventionalweapons.

50.50.50.50.50. According to the author, thegreatest military surprises in Americanhistory have been in Asia because

(1) The Americans failed toimplement their military strategiesmany miles away from theirown country.

(2) The Americans were unableto use their technologies likeintelligence satellites effectivelyto detect enemy movements.

(3) The Americans failed tounderstand the Asian culture ofwar that was based on stealth andsurprise.

(4) Clausewitz is inferior to Sun-tzu.

Page 20: Cat 1

20 MBA ENTRANCE EXAMINATION AT A GLANCE

Directions (Qs. Directions (Qs. Directions (Qs. Directions (Qs. Directions (Qs. 51 to 55) :51 to 55) :51 to 55) :51 to 55) :51 to 55) : In these In these In these In these In thesequestions, arrange the sentences A, B,questions, arrange the sentences A, B,questions, arrange the sentences A, B,questions, arrange the sentences A, B,questions, arrange the sentences A, B,C and D to forC and D to forC and D to forC and D to forC and D to form a logical sequencem a logical sequencem a logical sequencem a logical sequencem a logical sequencebetween sentences 1 and 6.between sentences 1 and 6.between sentences 1 and 6.between sentences 1 and 6.between sentences 1 and 6.

51.51.51.51.51. l. Making people laugh istricky.

A. At times, the intendedhumour may simply not comeoff.

B. Making people laugh whiletrying to sell them somethingis a tougher challenge, sincethe commercial can fall flaton two grounds.

C. There are many advertise-ments which do amuse butdo not even begin to set thecash tills ringing.

D. Again, it is rarely sufficientfor an advertiser simply toamuse the target audiencein order to reap the salesbenefit.

6. There are indications that insubstituting the hard sellfor a more entertainingapproach, some agencieshave rather thrown out thebaby with the bath water.

(1) CDBA (2) ABCD(3) BADC (4) DCBA

52.52.52.52.52. l. Picture a termite colony,occupying a tall mud humpon an African plain.

A. Hungry predators ofteninvade the colony andunsettle the balance.

B. The colony flourishes only ifthe proportion of soldiers toworkers remains roughly thesame, so that the queen andworkers can be protected bythe soldiers, and the queenand soldiers can be servicedby the workers.

C. But its fortunes are presentlyrestored, because theimmobile queen, walled inwell below ground level, layseggs not only in largeenough numbers, but also inthe varying proportionsrequired.

D. The hump is alive withworker termites and soldiertermites going about theirdistinct kinds of business.

6. How can we account for hermysterious ability to respondlike this to events on thedistant surface ?

(1) BADC (2) DBAC(3) ADCB (4) BDCA

53.53.53.53.53. l. According to recent research,the critical period fordeveloping language skills isbetween the ages of three andfive and a half years.

A. The read-to child already hasa large vocabulary and asense of grammar andsentence structure.

B. Children who are read to inthese years have a far betterchance of reading well inschool, indeed, of doing wellin all their subjects.

C. And the reason is actuallyquite simple.

D. This correlation is far andaway the highest yet foundbetween home influences andschool success.

6. Her comprehension oflanguage is therefore veryhigh.

(1)DACB (2) ADCB(3)ABCD (4) BDCA

Page 21: Cat 1

MBA ENTRANCE EXAMINATION AT A GLANCE 21

54.54.54.54.54. l. High-powered outboardmotors were considered to beone of the major threats tothe survival of the Belugawhales.

A. With these, hunters couldapproach Belugas withinhunting range and profit fromits inner skin and blubber.

B. To escape an approachingmotor, Belugas have learnedto dive into the ocean bottomand stay there for up to 20minutes, by which time theconfused predator has left.

C. Today, however, even withmuch more powerful engines,it is difficult to come close,because the whales seem todisappear suddenly just whenyou thought you had them inyour sights.

D. When the first outboardengines arrived in the early1930s, one came across 4and 8 HP motors.

6. Belugas seem to have usedtheir well-known sensitivity tonoise to evolve an‘avoidance’ strategy tooutsmart hunters and theirpowerful technologies.

(1) DACB (2) CDAB(3) ADBC (4) BDAC

55.55.55.55.55. l. The reconstruction of historyby post-revolutionary sciencetexts involves more than amultiplication of historicalmisconstructions.

A. Because they aim quickly toacquaint the student with whatthe contemporary scientificcommunity thinks it knows,textbooks treat the variousexperiments, concepts, laws

and theories of the currentnormal science as separatelyand as nearly seriatim aspossible.

B. Those misconstructions renderrevolutions invisible; thearrangement of the still visiblematerial in science textsimplies a process that, if itexisted, would denyrevolutions a function.

C. But when combined with thegenerally unhistorical air ofscience writing and with theoccasional systematicmisconstruction, one impres-sion is likely to follow.

D. As pedagogy this techniqueof presentation is unexcep-tionable.

6. Science has reached itspresent state by a series ofindividual discoveries andinventions that, whengathered together, constitutethe modern body of technicalknowledge.

(1) BADC (2) ADCB(3) DACB (4) CBDA

SECTION IISECTION IISECTION IISECTION IISECTION IIDirections (QDirections (QDirections (QDirections (QDirections (Qs. 56 to 74) :s. 56 to 74) :s. 56 to 74) :s. 56 to 74) :s. 56 to 74) : Answer Answer Answer Answer Answer

each of these questions independentlyeach of these questions independentlyeach of these questions independentlyeach of these questions independentlyeach of these questions independently.....56.56.56.56.56. The number of positive integer-

valued pairs (x, y) satisfying4x – 17y = 1 and x < 1000 is(1) 59 (2) 57 (3) 55 (4) 5857.57.57.57.57. Let a, b and c be the distinct

digits. Consider a two-digit number ‘ab’and a three-digit number ‘ccb’, bothdefined under the usual decimal numbersystem. If (ab)2 = ccb and ccb > 300,then the value of b is

(1) 1 (2) 0(3) 5 (4) 6

Page 22: Cat 1

22 MBA ENTRANCE EXAMINATION AT A GLANCE

58.58.58.58.58. The remainder when 784 isdivided by 342 is

(1) 0 (2) 1 (3) 49 (4) 34159.59.59.59.59. Ten points are marked on a

straight line and eleven pointsare marked on another straight line.How many triangles can be constructedwith vertices from among the abovepoints ?

(1) 495 (2) 550(3) 1045 (4) 247560.60.60.60.60. For a scholarship, at most n

candidates out of 2n + 1 can beselected. If the number of different waysof selection of atleast one candidate is63, the maximum number of candidatesthat can be selected for the scholarshipis

(1) 3 (2) 4 (3) 2 (4) 561.61.61.61.61. The speed of a railway engine is

42 Km per hour when no compartmentis attached and the reduction inspeed is directly proportional to thesquare root of the number ofcompartments attached. If the speed ofthe train carried by this engine is 24Km per hour when 9 compartments areattached, the maximum number ofcompartments that can be carried bythe engine is

(1) 49 (2) 48 (3) 46 (4) 4762.62.62.62.62. Total expenses of a boarding

house are partly fixed and partly varyinglinearly with the number of boarders.The average expenses per boarder isRs. 700 when there are 25 boardersand Rs. 600 when there are 50boarders. What is the average expensesper boarder when there are 100boarders ?

(1) 550 (2) 560(3) 540 (4) 53063.63.63.63.63. Forty percent of the employees of

a certain company are men and 75

percent of the men earn more thanRs. 25,000 per year. If 45 percent ofthe company’s employees earn morethan Rs. 25,000 per year, what fractionof the women employed by the companyearn Rs. 25,000 per year or less ?

(1) 112

(2) 41

(3) 31

(4) 43

64.64.64.64.64. If 116r =− and 812q2 =− ,what is the minimum possible value ofq/r ?

(1) –2/5 (2) 2/17(3) 10/17 (4) None of these65.65.65.65.65. If n = 1 + x, where x is the

product of four consecutive positiveintegers, then which of the followingis/are true ?

A. n is odd B. n is primeC. n is a perfect square

(1) A and C only(2) A and B only(3) A only(4) None of these66.66.66.66.66. In a survey of political preference,

78% of those asked were in favour ofatleast one of the proposals : I, II, III.50% of those asked favoured proposal I,30% favoured proposal II and 20%favoured proposal III. If 5% of thoseasked favoured all three of theproposals, what percentage of thoseasked favoured more than one of the 3proposals.

(1) 10 (2) 12 (3) 17 (4) 2267.67.67.67.67. For two positive integers a and

b, define the function hhhhh(a, b) as thegreatest common factor (gcf) of a, b.Let A be a set of n positive integers.G(A), the gcf of the elements of set Ais computed by repeatedly using thefunction hhhhh. The minimum number of timesfor which hhhhh is required to be used tocompute G is

(1) ½n (2) (n – 1)(3) n (4) None of these

Page 23: Cat 1

MBA ENTRANCE EXAMINATION AT A GLANCE 23

68.68.68.68.68. The figure below shows twoconcentric circles with centre O. PQRSis a square inscribed in the outer circle.It also circumscribes the inner circletouching it at points B, C, D and A.

What is the ratio of the perimeter of theouter circle to that of the polygon ABCD ?

(1) π/ 4 (2) 3π/ 2(3) π/ 2 (4) π69.69.69.69.69. Three labelled boxes containing

red and white cricket balls are allmislabelled. It is known that one of theboxes contains only white balls and oneonly red balls. The third contains amixture of red and white balls. You arerequired to correctly label the boxes withthe labels red, white and red & whiteby picking a sample of one ball fromonly one box. What is the label on thebox you should sample ?

(1) White(2) Red(3) Red and White(4) Not possible to determine from a

sample of one ball70.70.70.70.70. If n2 = 123456787654321, what

is n ?(1) 12344321 (2) 1235789(3) 11111111 (4) 111111171.71.71.71.71. Abraham, Border, Charlie, Dennis

and Elmer and their respective wivesrecently dined together and were seatedat a circular table. The seats were so

arranged that men and womenalternated and each woman was threeplaces distant from her husband.Mrs. Charlie sat to the left ofMr. Abraham. Mrs. Elmer sat two placesto the right of Mrs. Border. Who sat tothe right of Mr. Abraham ?

(1) Mrs. Dennis(2) Mrs. Elmer(3) Mrs. Border(4) Mrs. Border or Mrs. Dennis72.72.72.72.72. Navjivan Express from

Ahmedabad to Chennai leavesAhmedabad at 6 : 30 am and travels at50 km per hour towards Baroda situated100 km away. At 7:00 am, Howrah–Ahmedabad Express leaves Barodatowards Ahmedabad and travels at 40km per hour. At 7:30 am, Mr. Shah, thetraffic controller at Baroda, realises thatboth the trains are running on the sametrack. How much time does he have toavert a head-on collision between thetwo trains ?

(1) 15 minutes (2) 20 minutes(3) 25 minutes (4) 30 minutes73.73.73.73.73. There is a circle of radius 1 cm.

Each number of a sequence of regularpolygons Sl(n), n = 4, 5, 6, ...., wheren is the number of sides of the polygon,is circumscribing the circle; and eachmember of the sequence of regularpolygons S2(n), n = 4, 5, 6, ... wheren is the number of sides of the polygon,is inscribed in the circle. Let L1(n)and L2(n) denote the perimetersof the polygons S1(n) and S2(n)respectively.

Then {L1(13) + 2π} / L2(17) is(1) Greater than π/4 and less than 1(2) Greater than 1 and less than 2(3) Greater than 2(4) Less than π/4

A CO.

B

D RS

QP

Page 24: Cat 1

24 MBA ENTRANCE EXAMINATION AT A GLANCE

74.74.74.74.74. There is a square field with eachside 500 metres long. It has a compoundwall along its perimeter. At one of itscorners, a triangular area of the field isto be cordoned off by erecting a straightline fence. The compound wall and thefence will form its borders. If the lengthof the fence is 100 metres, what is themaximum area in square metres thatcan be cordoned off ?

(1) 2,500 (2) 10,000(3) 5,000 (4) 20,000Directions (Qs. 75 to 77) : Directions (Qs. 75 to 77) : Directions (Qs. 75 to 77) : Directions (Qs. 75 to 77) : Directions (Qs. 75 to 77) : TheseTheseTheseTheseThese

questions are based on the situationquestions are based on the situationquestions are based on the situationquestions are based on the situationquestions are based on the situationgiven below :given below :given below :given below :given below :

Ten coins are distributed among fourpeople P, Q, R, S such that one of themgets one coin, another gets two coins,the third gets three coins and the fourthgets four coins. It is known that Q getsmore coins than P and S gets fewercoins than R.

75.75.75.75.75. If the number of coins distributedto Q is twice the number of coinsdistributed to P, then which one of thefollowing is necessarily true ?

(1) R gets an even number of coins(2) R gets an odd number of coins(3) S gets an even number of coins(4) S gets an odd number of coins76.76.76.76.76. If R gets at least two more coins

than S, then which one of the followingis necessarily true ?

(1) Q gets atleast two more coinsthan S.

(2) Q gets more coins than P.(3) P gets more coins than S.(4) P and Q together get atleast five

coins.77.77.77.77.77. If Q gets fewer coins than R,

then which one of the following is notnecessarily true ?

(1) P and Q together get atleast fourcoins

(2) Q and S together get atleast fourcoins.

(3) R and S together get atleast fivecoins.

(4) P and R together get atleast fivecoins.

Directions (Qs. 78 to 80) :Directions (Qs. 78 to 80) :Directions (Qs. 78 to 80) :Directions (Qs. 78 to 80) :Directions (Qs. 78 to 80) : TheseTheseTheseTheseThesequestions are based on the situationquestions are based on the situationquestions are based on the situationquestions are based on the situationquestions are based on the situationgiven below :given below :given below :given below :given below :

A young girl Roopa leaves home withx flowers and goes to the bank of anearby river. On the bank of the river,there are four places of worship standingin a row. She dips all the x flowers intothe river. The number of flowers doubles.Then she enters the first place of worship,offers y flowers to the deity. She dips theremaining flowers into the river and againthe number of flowers doubles. She goesto the second place of worship and offersy flowers to the deity. She dips theremaining flowers into the river and againthe number of flowers doubles. She goesto the third place of worship and offersy flowers to the deity. She dips theremaining flowers into the river and againthe number of flowers doubles. She goesto the fourth place of worship and offersy flowers to the deity. Now she is leftwith no flowers in hand.

78.78.78.78.78. If Roopa leaves home with 30flowers, the number of flowers she offersto each deity is

(1) 30 (2) 31(3) 32 (4) 3379.79.79.79.79. The minimum number of flowers

that could be offered to each deity is(1) 0 (2) 15(3) 16 (4) Cannot be determined80.80.80.80.80. The minimum number of flowers

with which Roopa leaves home is(1) 16 (2) 15(3) 0 (4) Cannot be determined

Page 25: Cat 1

MBA ENTRANCE EXAMINATION AT A GLANCE 25

Directions (Qs. 81 and 82) :Directions (Qs. 81 and 82) :Directions (Qs. 81 and 82) :Directions (Qs. 81 and 82) :Directions (Qs. 81 and 82) : The The The The Thefollowing table presents the sweetnessfollowing table presents the sweetnessfollowing table presents the sweetnessfollowing table presents the sweetnessfollowing table presents the sweetnessof different items relative to sucrose,of different items relative to sucrose,of different items relative to sucrose,of different items relative to sucrose,of different items relative to sucrose,whose sweetness is taken to be 1.00.whose sweetness is taken to be 1.00.whose sweetness is taken to be 1.00.whose sweetness is taken to be 1.00.whose sweetness is taken to be 1.00.

LactoseLactoseLactoseLactoseLactose 0.160.160.160.160.16MaltoseMaltoseMaltoseMaltoseMaltose 0.320.320.320.320.32GlucoseGlucoseGlucoseGlucoseGlucose 0.740.740.740.740.74SucroseSucroseSucroseSucroseSucrose 1.001.001.001.001.00FructoseFructoseFructoseFructoseFructose 1.701.701.701.701.70SaccharinSaccharinSaccharinSaccharinSaccharin 675.00675.00675.00675.00675.0081.81.81.81.81. What is the minimum amount of

sucrose (to the nearest gram) that mustbe added to one-gram of saccharin tomake a mixture that will be at least 100times as sweet as glucose ?

(1) 7 (2) 8(3) 9 (4) 10082.82.82.82.82. Approximately how many times

sweeter than sucrose is a mixtureconsisting of glucose, sucrose andfructose in the ratio of 1 : 2 : 3 ?

(1) 1.3 (2) 1(3) 0.6 (4) 2.3Directions (Qs. 83 and 84) :Directions (Qs. 83 and 84) :Directions (Qs. 83 and 84) :Directions (Qs. 83 and 84) :Directions (Qs. 83 and 84) : These These These These These

questions are based on the situationquestions are based on the situationquestions are based on the situationquestions are based on the situationquestions are based on the situationgiven below :given below :given below :given below :given below :

A, B, C, D, E and F are a group offriends from a club. There are twohousewives, one lecturer, one architect,one accountant and one lawyer in thegroup. There are two married couplesin the group. The lawyer is married toD who is a housewife. No lady in thegroup is either an architect or anaccountant. C, the accountant, is marriedto F who is a lecturer. A is married toD and E is not a housewife.

83.83.83.83.83. What is E ?(1) Lawyer (2) Architect(3) Lecturer (4) Accountant84.84.84.84.84. How many members of the group

are males ?(1) 2 (2) 3(3) 4 (4) None of these

Directions (Qs. 85 and 86) : Directions (Qs. 85 and 86) : Directions (Qs. 85 and 86) : Directions (Qs. 85 and 86) : Directions (Qs. 85 and 86) : TheseTheseTheseTheseThesequestioquestioquestioquestioquestions are based on the situationns are based on the situationns are based on the situationns are based on the situationns are based on the situationgiven below :given below :given below :given below :given below :

Seven university cricket players areto be honoured at a special luncheon.The players will be seated on the daisalong one side of a single rectangulartable.

A and G have to leave the luncheonearly and must be seated at the extremeright end of the table which is closest tothe exit.

B will receive the Man of theMatch award and must be in the centerchair.

C and D, who are bitter rivals for theposition of wicket keeper, dislike eachother and should be seated as far apartas possible.

E and F are best friends and want tosit together.

85.85.85.85.85. Which of the followingmay not be seated at either end of thetable ?

(1) C (2) D(3) G (4) F86.86.86.86.86. Which of the following pairs may

not be seated together ?(1) E & A (2) B & D(3) C & F (4) G & DDirections (Qs. 87 and 88) : Directions (Qs. 87 and 88) : Directions (Qs. 87 and 88) : Directions (Qs. 87 and 88) : Directions (Qs. 87 and 88) : TheseTheseTheseTheseThese

questions are based on the situationquestions are based on the situationquestions are based on the situationquestions are based on the situationquestions are based on the situationgiven below :given below :given below :given below :given below :

A rectangle PRSU is divided into twosmaller rectangles PQTU and QRST bythe line TQ. PQ = 10 cm, QR = 5 cmand RS = 10 cm. Points A, B, F arewithin the rectangle PQTU and pointsC, D, E are within the rectangle QRST.The closest pair of points among thepairs (A, C), (A, D), (A, E), (F, C), (F, D),

(F, E), (B, C), (B, D), (B, E) are 310 cmapart.

Page 26: Cat 1

26 MBA ENTRANCE EXAMINATION AT A GLANCE

87.87.87.87.87. Which of the following statementsis necessarily true ?

(1) The closest pair of points amongthe six given points cannot be(F, C)

(2) Distance between A and B is greaterthan that between F and C

(3) The closest pair of points amongthe six given points is (C, D),(D, E) or (C, E)

(4) None of these88.88.88.88.88. AB > AF > BF; CD > DE > CE;

and BF 56= cm. Which is the closestpair of points among all the six givenpoints ?

(1) B, F (2) C, D(3) A, B (4) None of theseDirections (Q.s 89 to 92) : Directions (Q.s 89 to 92) : Directions (Q.s 89 to 92) : Directions (Q.s 89 to 92) : Directions (Q.s 89 to 92) : TheseTheseTheseTheseThese

questions are based on the situationquestions are based on the situationquestions are based on the situationquestions are based on the situationquestions are based on the situationgiven below :given below :given below :given below :given below :

In each of these questions a pair ofgraphs F(x) and F1(x) are given. Theseare composed of straight-line segments,shown as solid lines, in the domain

)2,2(x −∈ .If F1(x) = –F(x), choose your answer asa; if F1(x) = F(–x), choose your answeras b; if F1(x) = –F(–x), choose youranswer as c; and if none of the aboveis true, choose your answer as d.

89.89.89.89.89.

(1) a (2) b(3) c (4) d90.90.90.90.90.

(1) a (2) b(3) c (4) d91.91.91.91.91.

(1) a (2) b(3) c (4) d

F (x)

2

– 2 20

– 2

x

F (x)2

– 2 20

– 2

x

F1 (x)2

– 2 20

– 2

x

F (x)

2

– 2 20

– 2

xF1 (x)

2

– 2 20

– 2

x

F1 (x)

2

– 2 20

– 2

x

Page 27: Cat 1

MBA ENTRANCE EXAMINATION AT A GLANCE 27

92.92.92.92.92.

(1) a (2) b (3) c (4) dDirections (Qs. 93 and 94) :Directions (Qs. 93 and 94) :Directions (Qs. 93 and 94) :Directions (Qs. 93 and 94) :Directions (Qs. 93 and 94) : These These These These These

questions are based on the situationquestions are based on the situationquestions are based on the situationquestions are based on the situationquestions are based on the situationgiven begiven begiven begiven begiven below :low :low :low :low :

There are m blue vessels with knownvolumes v1, v2, ...., vm arranged inascending order of volumes where v1 >0.5 litre and vm < 1 litre. Each of theseis full of water, initially. The water fromeach of these is emptied into a minimumnumber of empty white vessels, each ofvolume 1 litre. The water from a bluevessel is not emptied into a white vesselunless the white vessel has enough emptyvolume to hold all the water of the bluevessel. The number of while vesselsrequired to empty all the blue vesselsaccording to the above rules was n.

93.93.93.93.93. Among the four values givenbelow, which is the least upper boundon e, where e is the total empty volumein the n white vessels at the end of theabove process ?

(1) m vm (2) m(1 – vm)(3) mv1 (4) m(1 – v1)94.94.94.94.94. Let the number of white vessels

needed be n1 for the emptying processdescribed above, if the volume of each

white vessel is 2 litres. Among thefollowing values, which is the least upperbound on n1 ?

(1) m/4(2) Smallest integer greater than or

equal to (n/2)(3) n(4) Greatest integer less than or equal

to (n/2)Directions (Qs. 95 to 97) :Directions (Qs. 95 to 97) :Directions (Qs. 95 to 97) :Directions (Qs. 95 to 97) :Directions (Qs. 95 to 97) : These These These These These

questions are based on the situationquestions are based on the situationquestions are based on the situationquestions are based on the situationquestions are based on the situationgiven below :given below :given below :given below :given below :

There are fifty integers a1, a2, ......,a50, not all of them necessarily different.Let the greatest integer of these fiftyintegers be referred to as G and thesmallest integer be referred to as L. Theintegers a1 through a24 form sequenceS1 and the rest form sequence S2. Eachmember of S1 is less than or equal toeach member of S2.

95.95.95.95.95. All values in S1 are changed insign while those in S2 remainunchanged. Which of the followingstatements is true ?

(1) Every member of S1 is greater thanor equal to every member of S2.

(2) G is in S1(3) If all numbers originally in S1 and

S2 had the same sign, then afterthe change of sign, the largestnumber of S1 and S2 is in S1.

(4) None of the above96.96.96.96.96. Elements of S1 are in ascending

order and those of S2 are in descendingorder. a24 and a25 are interchanged.Then, which of the following statements istrue ?

(1) S1 continues to be in ascendingorder

(2) S2 continues to be in descendingorder

(3) S1 continues to be in ascendingorder and S2 in descending order.

(4) None of the above

F1 (x)2

– 2 20

– 1

x

1

– 2

F (x)2

– 2 20

– 2

x– 1

1

1

Page 28: Cat 1

28 MBA ENTRANCE EXAMINATION AT A GLANCE

97.97.97.97.97. Every element of S1 is madegreater than or equal to every elementof S2 by adding to each element of S1an integer x. Then x cannot be less than

(1) 210

(2) The smallest value of S2(3) The largest value of S2(4) (G-L)Directions (Qs. 98 to 100) : Directions (Qs. 98 to 100) : Directions (Qs. 98 to 100) : Directions (Qs. 98 to 100) : Directions (Qs. 98 to 100) : TheseTheseTheseTheseThese

questions are based on the situationquestions are based on the situationquestions are based on the situationquestions are based on the situationquestions are based on the situationgiven below :given below :given below :given below :given below :

Let x and y be real numbers and letLet x and y be real numbers and letLet x and y be real numbers and letLet x and y be real numbers and letLet x and y be real numbers and letf(x, y) = | x + y |, F(f(x,y)) = –f(x + f(x, y) = | x + y |, F(f(x,y)) = –f(x + f(x, y) = | x + y |, F(f(x,y)) = –f(x + f(x, y) = | x + y |, F(f(x,y)) = –f(x + f(x, y) = | x + y |, F(f(x,y)) = –f(x + y)y)y)y)y)and G(f(x, y)) = –F(f(x,y))and G(f(x, y)) = –F(f(x,y))and G(f(x, y)) = –F(f(x,y))and G(f(x, y)) = –F(f(x,y))and G(f(x, y)) = –F(f(x,y))98.98.98.98.98. Which of the following statements

is true ?(1) F(f(x,y)) . G(f(x,y)) = –F(f(x,y)) . G(f(x,y))(2) F(f(x,y)) . G(f(x,y)) > –F(f(x,y)) . G(f(x,y))(3) F(f(x,y)) . G(f(x,y)) ≠ G(f(x,y)) . F(f(x,y))(4) F(f(x,y)) + G(f(x,y)) + f(x,y) = f(–x, –y)99.99.99.99.99. What is the value of f(G(f(1,0)),

f(F(f(1, 2)), G(f(1,2)))) ?(1) 3 (2) 2(3) 1 (4) 0100.100.100.100.100. Which of the following

expressions yields x2 as its result ?(1) F(f(x, –x)) . G(f(x, –x))(2) F(f(x,x)) . G(f(x,x)) . 4(3) –F(f(x,x) . G(f(x,x) ÷ log216(4) f(x, x) . f(x, x)Directions (Qs. 101 and 102) :Directions (Qs. 101 and 102) :Directions (Qs. 101 and 102) :Directions (Qs. 101 and 102) :Directions (Qs. 101 and 102) : These These These These These

questions are based on the situationquestions are based on the situationquestions are based on the situationquestions are based on the situationquestions are based on the situationgiven below given below given below given below given below :::::

A robot moves on a graph sheet withx and y axes. The robot is moved byfeeding it with a sequence of instructions.The different instructions that can be usedin moving it, along with their meaningsare as follows :

InstructionInstructionInstructionInstructionInstruction MeaniMeaniMeaniMeaniMeaningngngngngGOTO(x, y) Move to the point

with coordinates(x, y), no matter whereyou are currently

WALKX(p) Move parallel to thex-axis through adistance p in thepositive direction if pis positive and in thenegative direction ifp is negative

WALKY(p) Move parallel to they-axis through adistance p in thepositive direction if pis positive and in thenegative direction ifp is negative.

101.101.101.101.101. The robot reaches point (6, 6)when a sequence of three instructions isexecuted, the first of which is GOTO(x,y)instruction, the second is WALKX(2) andthe third is WALKY(4). What are thevalues of x and y ?

(1) 2, 4 (2) 0, 0(3) 4, 2 (4) 2, 2102.102.102.102.102. The robot is initially at (x, y),

x > 0 and y < 0. The minimum numberof instructions needed to be executed tobring it to the origin (0, 0) if you areprohibited from using the GOTOinstruction is

(1) 2 (2) 1(3) x + y (4) 0Directions (Qs. 103 to 105) :Directions (Qs. 103 to 105) :Directions (Qs. 103 to 105) :Directions (Qs. 103 to 105) :Directions (Qs. 103 to 105) : These These These These These

questions are based on the situationquestions are based on the situationquestions are based on the situationquestions are based on the situationquestions are based on the situationgiven below :given below :given below :given below :given below :

A road network (shown in the figurebelow) connects cities A, B, C and D.All road segments are straight lines. Dis the midpoint on the road connectingA and C. Roads AB and BC are at rightangles to each other with BC shorterthan AB. The segment AB is 100 kmlong.

Ms. X and Mr. Y leave A at 8.00 am,take different routes of city C and reachat the same time. X takes the highway

Page 29: Cat 1

MBA ENTRANCE EXAMINATION AT A GLANCE 29

from A to B to C and travels at anaverage speed of 61.875 km per hour.Y takes the direct route AC and travelsat 45 km per hour on segment AD. Y’sspeed on segment DC is 55 km perhour.

103.103.103.103.103. What is the average speed ofY in km per hour ?

(1) 47.5 (2) 49.5(3) 50 (4) 52104.104.104.104.104. The total distance travelled by Y

during the journey is approximately(1) 105 km (2) 150 km(3) 130 km (4) Cannot be determined105.105.105.105.105. What is the length of the road

segment BD ?(1) 50 km(2) 52.5 km(3) 55 km(4) Cannot be determinedDirections (Qs. 106 and 107) :Directions (Qs. 106 and 107) :Directions (Qs. 106 and 107) :Directions (Qs. 106 and 107) :Directions (Qs. 106 and 107) : These These These These These

questions are bquestions are bquestions are bquestions are bquestions are based on the situationased on the situationased on the situationased on the situationased on the situationgiven below :given below :given below :given below :given below :

Rajiv reaches city B from city A in 4hours driving at the speed of 35 km perhour for the first 2 hours and at 45 kmper hour for the next two hours. Aditifollows the same route but drives at threedifferent speeds : 30, 40 and 50 kmper hour covering an equal distance ineach speed segment. The two cars aresimilar with petrol consumptioncharacteristics (km per litre) shown inthe figure below.

106.106.106.106.106. The amount of petrol consumedby Aditi for the journey is

(1) 8.3 litres (2) 8.6 litres(3) 8.9 litres (4) 9.2 litres107.107.107.107.107. Zoheb would like to drive Aditi’s

car over the same route from A to Band minimise the petrol consumption forthe trip. The amount of petrol requiredby him is

(1) 6.67 litres (2) 7 litres(3) 6.33 litres (4) 6.0 litresDirections (Qs. 108 to 110) :Directions (Qs. 108 to 110) :Directions (Qs. 108 to 110) :Directions (Qs. 108 to 110) :Directions (Qs. 108 to 110) : TheseTheseTheseTheseThese

questions are based on the situationquestions are based on the situationquestions are based on the situationquestions are based on the situationquestions are based on the situationgiven below :given below :given below :given below :given below :

Recently, Ghosh Babu spent his wintervacation on Kyakya Island. During thevacation, he visited the local casinowhere he came across a new card game.Two players, using a normal deck of 52playing cards, play this game. Oneplayer is called the Dealer and the otheris called the Player. First, the Playerpicks a card at random from the deck.This is called the base card. The amountin rupees equal to the face value of thebase card is called the base amount.The face values of Ace, King, Queenand Jack are ten. For other cards, theface value is the number on the card.Once, the Player picks a card from thedeck, the Dealer pays him the baseamount. Then the Dealer picks a card

B C

D

AM

ileag

eKm

per

litre

24

16 16

504030Speed Km per hour

Page 30: Cat 1

30 MBA ENTRANCE EXAMINATION AT A GLANCE

from the deck and this card is called thetop card. If the top card is of the samesuit as the base card, the Player paystwice the base amount to the Dealer. Ifthe top card is of the same colour as thebase card (but not the same suit) then thePlayer pays the base amount to theDealer. If the top card happens to be ofa different colour than the base card, theDealer pays the base amount to thePlayer.

Ghosh Babu played the game4 times. First time he picked eightof clubs and Dealer picked queen ofclubs. Second time, he picked ten ofhearts and the Dealer picked two ofspades. Next time, Ghosh Babu pickedsix of diamonds and the Dealerpicked ace of hearts. Lastly, hepicked eight of spades and the Dealerpicked jack of spades. Answer thefollowing questions based on these fourgames.

108.108.108.108.108. If Ghosh Babu stopped playingthe game when his gain would bemaximised, the gain in Rupees wouldhave been

(1) 12 (2) 20(3) 16 (4) 4109. 109. 109. 109. 109. The initial money Ghosh

Babu had (before the beginning of thegame sessions) was Rs. X. At nopoint did he have to borrow any money.What is the minimum possible valueof X ?

(1) 16 (2) 8(3) 100 (4) 24110.110.110.110.110. If the final amount of money

that Ghosh Babu had with him wasRs. 100, what was the initial amount hehad with him ?

(1) 120 (2) 8(3) 4 (4) 96

SECTION IIISECTION IIISECTION IIISECTION IIISECTION IIIDirections (Qs. 111 to 120) : Directions (Qs. 111 to 120) : Directions (Qs. 111 to 120) : Directions (Qs. 111 to 120) : Directions (Qs. 111 to 120) : Each ofEach ofEach ofEach ofEach of

these questions consists of five statementsthese questions consists of five statementsthese questions consists of five statementsthese questions consists of five statementsthese questions consists of five statementsfollowed by options consisting of threefollowed by options consisting of threefollowed by options consisting of threefollowed by options consisting of threefollowed by options consisting of threestatements put together in a specific orstatements put together in a specific orstatements put together in a specific orstatements put together in a specific orstatements put together in a specific orderderderderder.....Choose the option which indicates aChoose the option which indicates aChoose the option which indicates aChoose the option which indicates aChoose the option which indicates avalid argument, that is, where the thirdvalid argument, that is, where the thirdvalid argument, that is, where the thirdvalid argument, that is, where the thirdvalid argument, that is, where the thirdstatement is a conclusion drawn from thestatement is a conclusion drawn from thestatement is a conclusion drawn from thestatement is a conclusion drawn from thestatement is a conclusion drawn from thepreceding two statements.preceding two statements.preceding two statements.preceding two statements.preceding two statements.

111.111.111.111.111.A. All software companies employ

knowledge workers.B. Tara Tech employs knowledge

workers.C. Tara Tech is a software company.D. Some software companies employ

knowledge workers.E. Tara Tech employs only

knowledge workers.(1) ABC (2) ACB(3) CDB (4) ACE

112.112.112.112.112.A. Traffic congestion increases carbon

monoxide in the environment.B. Increase in carbon monoxide is

hazardous to health.C. Traffic congestion is hazardous to

health.D. Some traffic congestion does

not cause increased carbonmonoxide.

E. Some traffic congestion is nothazardous to health.(1) CBA (2) BDE(3) CDE (4) BAC

113.113.113.113.113.A. Apples are not sweets.B. Some apples are sweet.C. All sweets are tasty.D. Some apples are not tasty.E. No apple is tasty.

(1) CEA (2) BDC(3) CBD (4) EAC

Page 31: Cat 1

MBA ENTRANCE EXAMINATION AT A GLANCE 31

114.114.114.114.114.A. Some towns in India are polluted.B. All polluted towns should be

destroyed.C. Town Meghana should be

destroyed.D. Town Meghana is polluted.E. Some towns in India should be

destroyed.(1) BDE (2) BAE(3) ADE (4) CDB

115.115.115.115.115.A. No patriot is a criminal.B. Bundledas is not a criminal.C. Bundledas is a patriot.D. Bogusdas is not a patriot.E. Bogusdas is a criminal.

(1) ACB (2) ABC(3) ADE (4) ABE

116.116.116.116.116.A. Ant eaters like ants.B. Boys are ant eaters.C. Balaram is an ant eater.D. Balaram likes ants.E. Balaram may eat ants.

(1) DCA (2) ADC(3) ABE (4) ACD

117.117.117.117.117.A. All actors are handsome.B. Some actors are popular.C. Ram is handsome.D. Ram is a popular actor.E. Some popular people are

handsome.(1) ACD (2) ABE(3) DCA (4) EDC

118.118.118.118.118.A. Modern industry is technology-

driven.B. BTI is a modern industry.C. BTI is technology-driven.D. BTI may be technology-driven.E. Technology-driven industry is

modern.(1) ABC (2) ABD(3) BCA (4) EBC

119.119.119.119.119.A. All Golmal islanders are blue-

coloured people.B. Some smart people are not blue-

coloured people.C. Some babies are blue-coloured.D. Some babies are smart.E. Some smart people are not

Golmal islanders.(1) BCD (2) ABE(3) CBD (4) None of these

120.120.120.120.120.A. MBAs are in great demand.B. Ram and Sita are in great demand.C. Ram is in great demand.D. Sita is in great demand.E. Ram and Sita are MBAs.

(1) ABE (2) ECD(3) AEB (4) EBA

Directions (Qs. 121 to 124) : Directions (Qs. 121 to 124) : Directions (Qs. 121 to 124) : Directions (Qs. 121 to 124) : Directions (Qs. 121 to 124) : EachEachEachEachEachof these questions has a main statementof these questions has a main statementof these questions has a main statementof these questions has a main statementof these questions has a main statementfollowed by four statements labelled A,followed by four statements labelled A,followed by four statements labelled A,followed by four statements labelled A,followed by four statements labelled A,B, C and D. Choose the ordered pair ofB, C and D. Choose the ordered pair ofB, C and D. Choose the ordered pair ofB, C and D. Choose the ordered pair ofB, C and D. Choose the ordered pair ofstatements where the first statementstatements where the first statementstatements where the first statementstatements where the first statementstatements where the first statementimplies the second, and the twoimplies the second, and the twoimplies the second, and the twoimplies the second, and the twoimplies the second, and the twostatements are logically consistent withstatements are logically consistent withstatements are logically consistent withstatements are logically consistent withstatements are logically consistent withthe main statement.the main statement.the main statement.the main statement.the main statement.

121. 121. 121. 121. 121. Either the orangutan is notangry, or he frowns upon the world.

A. The orangutan frowns upon theworld.

B. The orangutan is not angry.C. The orangutan does not frown

upon the world.D. The orangutan is angry.(1) CB only (2) DA only(3) AB only (4) CB and DA

122.122.122.122.122. Either Ravana is a demon, orhe is a hero.

A. Ravana is a hero.B. Ravana is a demon.C. Ravana is not a demon.D. Ravana is not a hero.(1) CD only (2) BA only(3) CD and BA (4) DB and CA

Page 32: Cat 1

32 MBA ENTRANCE EXAMINATION AT A GLANCE

123.123.123.123.123. Whenever Rajeev uses theinternet, he dreams about spiders.

A. Rajeev did not dream aboutspiders.

B. Rajeev used the internet.C. Rajeev dreamt about spiders.D. Rajeev did not use the internet.(1) AD (2) DC(3) CB (4) DA

124.124.124.124.124. If I talk to my professors,then I do not need to take a pill forheadache.

A. I talked to my professors.B. I did not need to take a pill

for headache.C. I needed to take a pill for

headache.D. I did not talk to my professors.(1) AB only (2) DC only(3) CD only (4) AB and CD

Directions (Qs. 125 to 134) : Directions (Qs. 125 to 134) : Directions (Qs. 125 to 134) : Directions (Qs. 125 to 134) : Directions (Qs. 125 to 134) : EachEachEachEachEachof these questions has a set of fourof these questions has a set of fourof these questions has a set of fourof these questions has a set of fourof these questions has a set of fourstatements. Each statement has threestatements. Each statement has threestatements. Each statement has threestatements. Each statement has threestatements. Each statement has threesegments. Choose the altersegments. Choose the altersegments. Choose the altersegments. Choose the altersegments. Choose the alternative whernative whernative whernative whernative whereeeeethe third segment in the statement canthe third segment in the statement canthe third segment in the statement canthe third segment in the statement canthe third segment in the statement canbe logically deduced using both thebe logically deduced using both thebe logically deduced using both thebe logically deduced using both thebe logically deduced using both thepreceding two, but not just from one ofpreceding two, but not just from one ofpreceding two, but not just from one ofpreceding two, but not just from one ofpreceding two, but not just from one ofthem.them.them.them.them.

125.125.125.125.125.A. No cowboys laugh. Some who

laugh are sphinxes. Somesphinxes are not cowboys.

B. All ghosts are fluorescent. Someghosts do not sing. Some singersare not fluorescent.

C. Cricketers indulge in swearing.Those who swear are hanged.Some who are hanged are notcricketers.

D. Some crazy people are pianists.All crazy people are whistlers.Some whistlers are pianists.(1) A and B (2) C only(3) A and D (4) D only

126.126.126.126.126.A. All good people are knights. All

warriors are good people. Allknights are warriors.

B. No footballers are ministers. Allfootballers are tough. Someministers are players.

C. All pizzas are snacks. Somemeals are pizzas. Some mealsare snacks.

D. Some barkers are musk-deer. Allbarkers are sloth bears. Somesloth bears are musk-deer.(1) C and D (2) B and C(3) A only (4) C only

127.127.127.127.127.A. Dinosaurs are pre-historic

creatures. Water-buffaloes are notdinosaurs. Water- buffaloes arenot pre-historic creatures.

B. All politicians are frank. No frankpeople are crocodiles. Nocrocodiles are politicians.

C. No diamond is quartz. No opalis quartz. Diamonds are opals.

D. All monkeys like bananas. SomeGI Joes like bananas. Some GIJoes are monkeys.(1) C only (2) B only(3) A and D (4) B and C

128.128.128.128.128.A. All earthquakes cause havoc.

Some landslides cause havoc.Some ear thquakes causelandslides.

B. All glass things are transparent.Some curios are glass things.Some curios are transparent.

C. All clay objects are brittle.All XY are clay objects.Some XY are brittle.

D. No criminal is a patriot.Ram is not a patriot.Ram is a criminal.(1) D only (2) B only(3) C and B (4) A only

Page 33: Cat 1

MBA ENTRANCE EXAMINATION AT A GLANCE 33

129129129129129.....A. MD is an actor. Some actors are

pretty. MD is pretty.B. Some men are cops. All cops

are brave. Some brave peopleare cops.

C. All cops are brave. Some menare cops. Some men are brave.

D. All actors are pretty; MD is notan actor; MD is not pretty.(1) D only (2) C only(3) A only (4) B and C

130.130.130.130.130.A. All IIMs are in India.

No BIMs are in India.No IIMs are BIMs.

B. All IIMs are in India.No BIMs are in India.No BIMs are IIMs.

C. Some IIMs are not in India.Some BIMs are not in India.Some IIMs are BIMs.

D. Some IIMs are not in India.Some BIMs are not in India.Some BIMs are IIMs.(1) A and B (2) C and D(3) A only (4) B only

131.131.131.131.131.A. Citizens of Yes Islands speak only

the truth. Citizens of Yes Islandsare young people. Young peoplespeak only the truth.

B. Citizens of Yes Islands speak onlythe truth. Some Yes Islands arein the Atlantic. Some citizens ofYes Islands are in the Atlantic.

C. Citizens of Yes Islands speak onlythe truth. Some young people arecitizens of Yes Islands. Someyoung people speak only thetruth.

D. Some people speak onlythe truth. Some citizens ofYes Islands speak only thetruth. Some people who speakonly the truth are citizens ofYes Islands.(1) A only (2) B only(3) C only (4) D only

132.132.132.132.132.A. All mammals are viviparous.

Some fish are viviparous.Some fish are mammals.

B. All birds are oviparous.Some fish are not oviparous.Some fish are birds.

C. No mammal is oviparous. Somecreatures are oviparous andsome are not. Some creatures arenot mammals.

D. Some creatures are mammals.Some creatures are viviparous.Some mammals are viviparous.(1) A only (2) B only(3) C only (4) D only

133.133.133.133.133.A. Many singers are not writers.

All poets are singers.Some poets are not writers.

B. Giants climb beanstalks.Some chicken do not climbbeanstalks.Some chicken are not giants.

C. All explorers live in snowdrifts.Some penguins live in snowdrifts.Some penguins are explorers.

D. Amar is taller than Akbar.Anthony is shorter than Amar.Akbar is shorter than Anthony.(1) A only (2) B only(3) B and C (4) D only

Page 34: Cat 1

34 MBA ENTRANCE EXAMINATION AT A GLANCE

134.134.134.134.134.A. A few farmers are rocket

scientists. Some rocket scientistscatch snakes. A few farmerscatch snakes.

B. Poonam is a kangaroo. Somekangaroos are made of teak.Poonam is made of teak.

C. No bulls eat grass. All matadorseat grass. No matadors arebulls.

D. Some skunks drive Cadillacs. Allskunks are polar bears. Somepolar bears drive Cadillacs.(1) B only(2) A and C(3) C only(4) C and D

Directions (Qs. 135 to 138) : Directions (Qs. 135 to 138) : Directions (Qs. 135 to 138) : Directions (Qs. 135 to 138) : Directions (Qs. 135 to 138) : TheseTheseTheseTheseThesequestions are based on the situationquestions are based on the situationquestions are based on the situationquestions are based on the situationquestions are based on the situationgiven below :given below :given below :given below :given below :

The figure below presents salesand net profit, in Rs. Crore, of IVPLtd for five years from 1994-95to 1998-99. During this period, thesales increased from Rs. 100 croreto Rs. 680 crore. Correspondingly,the net profit increased from Rs. 2.2crore to Rs. 12 crore. Net profitis defined as the excess of sales overtotal costs.

135.135.135.135.135. The highest percentage ofgrowth in sales, relative to the previousyear, occurred in

(1) 1995-96 (2) 1996-97(3) 1997-98 (4) 1998-99136.136.136.136.136. The highest percentage growth

in net profit, relative to the previous year,was achieved in

(1) 1998-99 (2) 1997-98(3) 1996-97 (4) 1995-96137.137.137.137.137. Defining profitability as the ratio

of net profit to sales, IVP Ltd. recordedthe highest profitability in

(1) 1998-99 (2) 1997-98(3) 1994-95 (4) 1996-97138.138.138.138.138. With profitability as defined in

Q. 137, it can be concluded that(1) Profitability is non-decreasing

during the five years from1994-95 to 1998-99.

(2) Profitability is non-increasingduring the five years from1994-95 to 1998-99.

(3) Profitability remained constantduring the five years from1994-95 to 1998-99.

(4) None of the aboveDirections (Qs. 139 to 144) :Directions (Qs. 139 to 144) :Directions (Qs. 139 to 144) :Directions (Qs. 139 to 144) :Directions (Qs. 139 to 144) :

Consider the inforConsider the inforConsider the inforConsider the inforConsider the information prmation prmation prmation prmation providedovidedovidedovidedovidedin the figure below relating toin the figure below relating toin the figure below relating toin the figure below relating toin the figure below relating toIndia’India’India’India’India’s fors fors fors fors foreign trade in 1997-98eign trade in 1997-98eign trade in 1997-98eign trade in 1997-98eign trade in 1997-98and the first eight months of 1998-99.and the first eight months of 1998-99.and the first eight months of 1998-99.and the first eight months of 1998-99.and the first eight months of 1998-99.TTTTTotal trade with a rotal trade with a rotal trade with a rotal trade with a rotal trade with a region is definedegion is definedegion is definedegion is definedegion is definedas the sum of exporas the sum of exporas the sum of exporas the sum of exporas the sum of expor ts to andts to andts to andts to andts to andimpoimpoimpoimpoimporrrrr ts frts frts frts frts from that rom that rom that rom that rom that region. Tegion. Tegion. Tegion. Tegion. Traderaderaderaderadedeficit is defined as the excess ofdeficit is defined as the excess ofdeficit is defined as the excess ofdeficit is defined as the excess ofdeficit is defined as the excess ofimporimporimporimporimports over exports over exports over exports over exports over exports. Tts. Tts. Tts. Tts. Trade deficitrade deficitrade deficitrade deficitrade deficitmay be negative.may be negative.may be negative.may be negative.may be negative.

AAAAA ::::: U.S.A.U.S.A.U.S.A.U.S.A.U.S.A. BBBBB ::::: GerGerGerGerGermanymanymanymanymanyCCCCC ::::: Other E.U.Other E.U.Other E.U.Other E.U.Other E.U. DDDDD ::::: U. K.U. K.U. K.U. K.U. K.EEEEE ::::: JapanJapanJapanJapanJapan FFFFF ::::: RussiaRussiaRussiaRussiaRussiaGGGGG ::::: Other East EuropeOther East EuropeOther East EuropeOther East EuropeOther East EuropeHHHHH ::::: OPECOPECOPECOPECOPEC IIIII ::::: AsiaAsiaAsiaAsiaAsiaJJJJJ ::::: Other L.D.CsOther L.D.CsOther L.D.CsOther L.D.CsOther L.D.CsKKKKK ::::: OthersOthersOthersOthersOthers

700600500400300200100

0

14121086420

94-5 95-6 96-7 97-8 98-9

Page 35: Cat 1

MBA ENTRANCE EXAMINATION AT A GLANCE 35

SourSourSourSourSource of Imporce of Imporce of Imporce of Imporce of Importststststs1997-98 : Impor1997-98 : Impor1997-98 : Impor1997-98 : Impor1997-98 : Imports into India : $40779 millionts into India : $40779 millionts into India : $40779 millionts into India : $40779 millionts into India : $40779 million

1998-991998-991998-991998-991998-99 : : : : : Impor Impor Impor Impor Impor ts into India (April-Novts into India (April-Novts into India (April-Novts into India (April-Novts into India (April-Nov.) : $28126 million.) : $28126 million.) : $28126 million.) : $28126 million.) : $28126 million

Destination of ExporDestination of ExporDestination of ExporDestination of ExporDestination of Exportststststs1997-98 : Expor1997-98 : Expor1997-98 : Expor1997-98 : Expor1997-98 : Exports frts frts frts frts from India : $33979 millionom India : $33979 millionom India : $33979 millionom India : $33979 millionom India : $33979 million

1998-99 : Expor1998-99 : Expor1998-99 : Expor1998-99 : Expor1998-99 : Exports frts frts frts frts from India (April-Novom India (April-Novom India (April-Novom India (April-Novom India (April-Nov.) : $21436 million.) : $21436 million.) : $21436 million.) : $21436 million.) : $21436 million

139.139.139.139.139. What is the region with whichIndia had the highest total trade in1997-98 ?

1. USA 2. Other E. U.3. OPEC 4. Others140.140.140.140.140. In 1997-98 the amount of Indian

exports, in millions US $, to the regionwith which India had the lowest totaltrade, is approximately

1. 750 2. 3403. 220 4. 440141.141.141.141.141. In 1997-98, the trade deficit

with respect to India, in billions of US $, forthe region with the highest trade deficitwith respect to India is approximatelyequal to

1. 6.0 2. 3.03. 4.5 4. 7.5142.142.142.142.142. What is the region with the

lowest trade deficit with India in1997-98 ?

1. USA 2. Asia3. Others 4. Other E. U.Directions (Qs. 143 and 144)Directions (Qs. 143 and 144)Directions (Qs. 143 and 144)Directions (Qs. 143 and 144)Directions (Qs. 143 and 144) ::::: TheseTheseTheseTheseThese

questions are based on the situationquestions are based on the situationquestions are based on the situationquestions are based on the situationquestions are based on the situationgiven below :given below :given below :given below :given below :

Assume that the average monthlyexports from India and imports to Indiaduring the remaining four months of1998-99 would be the same as that forthe first eight months of the year.

143.143.143.143.143. What is the region to whichIndian exports registered the highestpercentage growth between 1997-98and 1998-99 ?

(1) Other East Europe(2) USA(3) Asia(4) Exports have declined, no growth144.144.144.144.144. What is the percentage growth

rate in India’s total trade deficit between1997-98 and 1998-99 ?

(1) 43 (2) 47(3) 50 (4) 40

J5%

K1%

A9% B

5%

C13%

D6%

E6%

G21%

F1%

H17%

I16%

J4%

K1% A

9% B5%

C12%

D6%

E5%

G19%

F2%

H23%

I14%

J5%

K1%

A19%

B6%

C14%

D6%E

6%G

10%

F3%

H10%

I20%

J5%

K1%

A23%

B5%

C14%

D5%

E5%

G12%

F2%

H10%

I18%

Page 36: Cat 1

36 MBA ENTRANCE EXAMINATION AT A GLANCE

Directions (Qs. 145 to 148) : Directions (Qs. 145 to 148) : Directions (Qs. 145 to 148) : Directions (Qs. 145 to 148) : Directions (Qs. 145 to 148) : TheseTheseTheseTheseThesequestions are based on the pricequestions are based on the pricequestions are based on the pricequestions are based on the pricequestions are based on the pricefluctuations of four commodities—arfluctuations of four commodities—arfluctuations of four commodities—arfluctuations of four commodities—arfluctuations of four commodities—arharharharharhar,,,,,pepperpepperpepperpepperpepper, sugar and gold during, sugar and gold during, sugar and gold during, sugar and gold during, sugar and gold duringFebrFebrFebrFebrFebruaruaruaruaruary–July 1999 as described in they–July 1999 as described in they–July 1999 as described in they–July 1999 as described in they–July 1999 as described in thefigure below :figure below :figure below :figure below :figure below :

Arhar (Friday quotations)Arhar (Friday quotations)Arhar (Friday quotations)Arhar (Friday quotations)Arhar (Friday quotations)

Pepper (Friday quotations)Pepper (Friday quotations)Pepper (Friday quotations)Pepper (Friday quotations)Pepper (Friday quotations)

Sugar (Friday quotations)Sugar (Friday quotations)Sugar (Friday quotations)Sugar (Friday quotations)Sugar (Friday quotations)

Gold (Friday quotations)Gold (Friday quotations)Gold (Friday quotations)Gold (Friday quotations)Gold (Friday quotations)

145.145.145.145.145. Price change of a commodityis defined as the absolute difference inending and beginning prices expressedas a percentage of the beginning. Whatis the commodity with the highest pricechange ?

1. Arhar 2. Pepper3. Sugar 4. Gold146.146.146.146.146. Price Volatility (PV) of a

commodity is defined as follows :PV = (Highest price during

the period – Lowest price during theperiod) / Average price during theperiod.

What is the commodity with theLowest Price Volatility ?

1. Arhar 2. Pepper3. Sugar 4. Gold147.147.147.147.147. Mr. X, a funds manager with an

investment company, invested 25% ofhis funds in each of the four commoditiesat the beginning of the period. He soldthe commodities at the end of the period.His investments in the commoditiesresulted in

1. 17% profit 2. 5.5% loss3. No profit, no loss 4. 4.3% profit148.148.148.148.148. The price volatility of the

commodity with the highest PV duringthe February-July period is approximatelyequal to

1. 3% 2. 40%3. 20% 4. 12%Directions (Qs. 149 to 153) :Directions (Qs. 149 to 153) :Directions (Qs. 149 to 153) :Directions (Qs. 149 to 153) :Directions (Qs. 149 to 153) :

These questions are based on theThese questions are based on theThese questions are based on theThese questions are based on theThese questions are based on thetable below prtable below prtable below prtable below prtable below presenting dataesenting dataesenting dataesenting dataesenting dataon percentage population coveredon percentage population coveredon percentage population coveredon percentage population coveredon percentage population coveredby drinking water and sanitationby drinking water and sanitationby drinking water and sanitationby drinking water and sanitationby drinking water and sanitationfacilities in selected Asian countries.facilities in selected Asian countries.facilities in selected Asian countries.facilities in selected Asian countries.facilities in selected Asian countries.

Country A is said to dominateB or A > B if A has higher percent-age in total coverage for bothdrinking water and sanitation facilities

WWWWWeeks (1999)eeks (1999)eeks (1999)eeks (1999)eeks (1999)

WWWWWeeks (1999)eeks (1999)eeks (1999)eeks (1999)eeks (1999)

WWWWWeeks (1999)eeks (1999)eeks (1999)eeks (1999)eeks (1999)

WWWWWeeks (1999)eeks (1999)eeks (1999)eeks (1999)eeks (1999)

Page 37: Cat 1

MBA ENTRANCE EXAMINATION AT A GLANCE 37

and B is said to be dominated by Aor B < A.

A country is said to be on thecoverage frontier if no other countrydominates it. Similarly, a country is noton the coverage frontier if it is dominatedby atleast one other country.

149.149.149.149.149. What are the countries on thecoverage frontier ?

(1) India and China(2) Sri Lanka and Indonesia(3) Philippines and Bangladesh(4) Nepal and Pakistan150.150.150.150.150. Which of the following

statements are true ?A. India > Pakistan & India >

IndonesiaB. India > China & India > NepalC. Sri Lanka > ChinaD. China > Nepal(1) A and C (2) B and D(3) A, B and C (4) B, C and D151.151.151.151.151. Using only the data presented

under Sanitation Facilities columns, it canbe concluded that rural population inIndia as a percentage of its totalpopulation is approximately

(1) 76 (2) 70(3) 73 (4) Cannot be determined152.152.152.152.152. Again, using only the data

presented under Sanitation Facilitiescolumns, sequence China, Indonesia andPhilippines in ascending order of ruralpopulation as a percentage of theirrespective total populations. The correctorder is

(1) Philippines, Indonesia, China(2) Indonesia, China, Philippines(3) Indonesia, Philippines, China(4) China, Indonesia, Philippines153.153.153.153.153. India is not on the coverage

frontier becauseA. It is lower than Bangladesh in

terms of coverage of drinkingwater facilities.

B. It is lower than Sri Lanka interms of coverage of sanitationfacilities.

C. It is lower than Pakistan interms of coverage of sanitationfacilities.

D. It is dominated by Indonesia.(1) A and B (2) A and C(3) D (4) None of these

Population CoverPopulation CoverPopulation CoverPopulation CoverPopulation Covered by Drinking Wed by Drinking Wed by Drinking Wed by Drinking Wed by Drinking Water and Sanitation Facilitiesater and Sanitation Facilitiesater and Sanitation Facilitiesater and Sanitation Facilitiesater and Sanitation FacilitiesPercentage Coverage

Drinking WDrinking WDrinking WDrinking WDrinking Wateraterateraterater Sanitation FacilitiesSanitation FacilitiesSanitation FacilitiesSanitation FacilitiesSanitation FacilitiesUrbanUrbanUrbanUrbanUrban RuralRuralRuralRuralRural TTTTTotalotalotalotalotal UrbanUrbanUrbanUrbanUrban RuralRuralRuralRuralRural TTTTTotalotalotalotalotal

India 85 79 81 70 14 29Bangladesh 99 96 97 79 44 48China 97 56 67 74 7 24Pakistan 82 69 74 77 22 47Philippines 92 80 86 88 66 77Indonesia 79 54 62 73 40 51Sri Lanka 88 52 57 68 62 63Nepal 88 60 63 58 12 18

Source : World Resources 1998-99, p. 251, UNDP, UNEP and World Bank

Page 38: Cat 1

38 MBA ENTRANCE EXAMINATION AT A GLANCE

Directions (Qs. 15Directions (Qs. 15Directions (Qs. 15Directions (Qs. 15Directions (Qs. 154 and 155) : 4 and 155) : 4 and 155) : 4 and 155) : 4 and 155) : TheseTheseTheseTheseThesequestions relate to the above table withquestions relate to the above table withquestions relate to the above table withquestions relate to the above table withquestions relate to the above table withthe additional proviso that the gap beweenthe additional proviso that the gap beweenthe additional proviso that the gap beweenthe additional proviso that the gap beweenthe additional proviso that the gap beweenthe population coverages of sanitationthe population coverages of sanitationthe population coverages of sanitationthe population coverages of sanitationthe population coverages of sanitationfacilities and drinking water facilities is afacilities and drinking water facilities is afacilities and drinking water facilities is afacilities and drinking water facilities is afacilities and drinking water facilities is ameasure of disparity in coverage.measure of disparity in coverage.measure of disparity in coverage.measure of disparity in coverage.measure of disparity in coverage.

154.154.154.154.154. The country with the mostdisparity in coverage of rural sector is

(1) India (2) Bangladesh(3) Nepal (4) None of these155.155.155.155.155. The country with the least

disparity in coverage of urban sector is(1) India (2) Pakistan(3) Philippines (4) None of theseDirections (Qs. 156 to 165) :Directions (Qs. 156 to 165) :Directions (Qs. 156 to 165) :Directions (Qs. 156 to 165) :Directions (Qs. 156 to 165) : Each Each Each Each Each

of these questions is followed byof these questions is followed byof these questions is followed byof these questions is followed byof these questions is followed bytwo statements, A and B. Answertwo statements, A and B. Answertwo statements, A and B. Answertwo statements, A and B. Answertwo statements, A and B. Answereach question using the followingeach question using the followingeach question using the followingeach question using the followingeach question using the followinginstructions :instructions :instructions :instructions :instructions :

Choose 1 if the question can beChoose 1 if the question can beChoose 1 if the question can beChoose 1 if the question can beChoose 1 if the question can beanswered by using one of the statementsanswered by using one of the statementsanswered by using one of the statementsanswered by using one of the statementsanswered by using one of the statementsalone, but cannot be answered usingalone, but cannot be answered usingalone, but cannot be answered usingalone, but cannot be answered usingalone, but cannot be answered usingthe other statement alone.the other statement alone.the other statement alone.the other statement alone.the other statement alone.

Choose 2 if the question can beChoose 2 if the question can beChoose 2 if the question can beChoose 2 if the question can beChoose 2 if the question can beanswered by using either statementanswered by using either statementanswered by using either statementanswered by using either statementanswered by using either statementalone.alone.alone.alone.alone.

Choose 3 if the question can beChoose 3 if the question can beChoose 3 if the question can beChoose 3 if the question can beChoose 3 if the question can beanswered by using both statementsanswered by using both statementsanswered by using both statementsanswered by using both statementsanswered by using both statementstogethertogethertogethertogethertogether, but cannot be answer, but cannot be answer, but cannot be answer, but cannot be answer, but cannot be answered usinged usinged usinged usinged usingeither statement alone.either statement alone.either statement alone.either statement alone.either statement alone.

Choose 4 if the question cannot beChoose 4 if the question cannot beChoose 4 if the question cannot beChoose 4 if the question cannot beChoose 4 if the question cannot beanswered even by using both statementsanswered even by using both statementsanswered even by using both statementsanswered even by using both statementsanswered even by using both statementstogethertogethertogethertogethertogether.....

156.156.156.156.156. The average weight of studentsin a class is 50 kg. What is the numberof students in the class ?

A. The heaviest and the lightestmembers of the class weigh 60kg and 40 kg respectively.

B. Exclusion of the heaviest and thelightest members from the classdoes not change the averageweight of students.

157.157.157.157.157. A small storage tank is sphericalin shape. What is the storage volumeof the tank ?

A. The wall thickness of the tank is1 cm.

B. When the empty spherical tank isimmersed in a large tank filled withwater, 20 litres of water overflowfrom the large tank.

158.158.158.158.158. Mr. X starts walking northwardsalong the boundary of a field from pointA on the boundary and after walkingfor 150 metres reaches B, and thenwalks westwards again along theboundary for another 100 metres whenhe reaches C. What is the maximumdistance between any pair of points onthe boundary of the field ?

A. The field is rectangular in shape.B. The field is a polygon with C as

one of its vertices and A the mid-point of a side.

159.159.159.159.159. A line graph on a graph sheetshows the revenue for each year from1990 through 1998 by points and joinsthe successive points by straight linesegments. The point for revenue of 1990is labelled A, that for 1991 as B, andthat for 1992 as C. What is the ratio ofthe growth in revenue between 1991-92 and 1990-91 ?

A. The angle between AB and X-axiswhen measured with a protractoris 40 degrees and the anglebetween CB and the X-axis is 80degrees.

B. The scale of Y-axis is 1 cm = Rs. 1000160.160.160.160.160. There is a circle with centre C

at the origin and radius r cm. Twotangents are drawn from an externalpoint D at a distance d cm from thecentre. What are the angles betweeneach tangent and X-axis ?

A. The coordinates of D are givenB. The X-axis bisects one of the

tangents.

Page 39: Cat 1

MBA ENTRANCE EXAMINATION AT A GLANCE 39

161.161.161.161.161. Find a pair of real numbers x andy that satisfy the following two equationssimultaneously. It is known that the valuesof a, b, c, d, e and f are non-zero.

ax + by = cdx + ey = f

A. a = kd and b = ke, c = kf, k ≠ 0B. a = b = 1, d = e = 2, f ≠ 2c162.162.162.162.162. Three professors A, B and C

are separately given three sets ofnumbers to add. They were expected tofind the answers to 1 + 1, 1 + 1 + 2,and 1 + 1 respectively. Their respectiveanswers were 3, 3 and 2. How manyof the professors are mathematicians ?

A. A mathematician can never addtwo numbers correctly, but canalways add three numbers correctly.

B. When a mathematician makes amistake in a sum, the error is +1or –1.

163.163.163.163.163. How many among the fourstudents A, B, C and D have passed theexam ?

A. The following is a true statement :A and B passed the exam.

B. The following is a false statement :At least one among C and D haspassed the exam.

164.164.164.164.164. What is the distance x between twocities A and B in integral number of Km ?

A. x satisfies the equationlog2x x=

B. x < 10 Km165.165.165.165.165. Mr. Mendel grew one hundred

flowering plants from black seeds andwhite seeds, each seed giving rise toone plant. A plant gives flowers of onlyone colour. From a black seed comes aplant giving red or blue flowers. From awhite seed comes a plant giving red orwhite flowers. How many black seedswere used by Mr. Mendel ?

A. The number of plants with whiteflowers was 10.

B. The number of plants with redflowers was 70.

ANSWERSANSWERSANSWERSANSWERSANSWERS1.1.1.1.1. (1) 2.2.2.2.2. (4) 3.3.3.3.3. (2) 4.4.4.4.4. (1)5.5.5.5.5. (3) 6.6.6.6.6. (3) 7.7.7.7.7. (2) 8.8.8.8.8. (3)9.9.9.9.9. (4) 10.10.10.10.10. (2) 11.11.11.11.11. (3) 12.12.12.12.12. (1)

13.13.13.13.13. (2) 14.14.14.14.14. (4) 15.15.15.15.15. (1) 16.16.16.16.16. (4)17.17.17.17.17. (1) 18.18.18.18.18. (3) 19.19.19.19.19. (2) 20.20.20.20.20. (1)21.21.21.21.21. (2) 22.22.22.22.22. (4) 23.23.23.23.23. (3) 24.24.24.24.24. (2)25.25.25.25.25. (2) 26.26.26.26.26. (1) 27.27.27.27.27. (4) 28.28.28.28.28. (3)29.29.29.29.29. (4) 30.30.30.30.30. (4) 31.31.31.31.31. (3) 32.32.32.32.32. (2)33.33.33.33.33. (3) 34.34.34.34.34. (1) 35.35.35.35.35. (3) 36.36.36.36.36. (2)37.37.37.37.37. (4) 38.38.38.38.38. (1) 39.39.39.39.39. (2) 40.40.40.40.40. (1)41.41.41.41.41. (4) 42.42.42.42.42. (4) 43.43.43.43.43. (2) 44.44.44.44.44. (3)45.45.45.45.45. (4) 46.46.46.46.46. (2) 47.47.47.47.47. (1) 48.48.48.48.48. (4)49.49.49.49.49. (1) 50.50.50.50.50. (3) 51.51.51.51.51. (3) 52.52.52.52.52. (2)53.53.53.53.53. (4) 54.54.54.54.54. (1) 55.55.55.55.55. (1) 56.56.56.56.56. (1)57.57.57.57.57. (1) 58.58.58.58.58. (2) 59.59.59.59.59. (3) 60.60.60.60.60. (1)61.61.61.61.61. (2) 62.62.62.62.62. (1) 63.63.63.63.63. (4) 64.64.64.64.64. (4)65.65.65.65.65. (1) 666666.6.6.6.6. (3) 67.67.67.67.67. (2) 68.68.68.68.68. (3)69.69.69.69.69. (3) 70.70.70.70.70. (3) 71.71.71.71.71. (4) 72.72.72.72.72. (2)73.73.73.73.73. (3) 74.74.74.74.74. (1) 75.75.75.75.75. (4) 76.76.76.76.76. (2)77.77.77.77.77. (1) 78.78.78.78.78. (3) 79.79.79.79.79. (3) 80.80.80.80.80. (2)81.81.81.81.81. (3) 82.82.82.82.82. (1) 83.83.83.83.83. (2) 84.84.84.84.84. (2)85.85.85.85.85. (4) 86.86.86.86.86. (1) 87.87.87.87.87. (1) 88.88.88.88.88. (4)89.89.89.89.89. (4) 90.90.90.90.90. (2) 91.91.91.91.91. (2) 92.92.92.92.92. (3)93.93.93.93.93. (4) 94.94.94.94.94. (2) 95.95.95.95.95. (4) 96.96.96.96.96. (1)97.97.97.97.97. (4) 98.98.98.98.98. (4) 99.99.99.99.99. (3) 100.100.100.100.100. (3)

101.101.101.101.101. (3) 102.102.102.102.102. (1) 103.103.103.103.103. (2) 104.104.104.104.104. (1)105.105.105.105.105. (2) 106.106.106.106.106. (3) 107.107.107.107.107. (1) 108.108.108.108.108. (1)109.109.109.109.109. (2) 110.110.110.110.110. (4) 111.111.111.111.111. (2) 112.112.112.112.112. (4)113.113.113.113.113. (1) 114.114.114.114.114. (2) 115.115.115.115.115. (1) 116.116.116.116.116. (4)117.117.117.117.117. (2) 118.118.118.118.118. (1) 119.119.119.119.119. (2) 120.120.120.120.120. (3)121.121.121.121.121. (4) 122.122.122.122.122. (4) 123.123.123.123.123. (1) 124.124.124.124.124. (4)125.125.125.125.125. (3) 126.126.126.126.126. (1) 127.127.127.127.127. (2) 128.128.128.128.128. (3)129.129.129.129.129. (2) 130.130.130.130.130. (1) 131.131.131.131.131. (3) 132.132.132.132.132. (3)133.133.133.133.133. (2) 134.134.134.134.134. (4) 135.135.135.135.135. (1) 136.136.136.136.136. (4)137.137.137.137.137. (2) 138.138.138.138.138. (4) 139.139.139.139.139. (3) 140.140.140.140.140. (2)141.141.141.141.141. (1) 142.142.142.142.142. (1) 143.143.143.143.143. (2) 144.144.144.144.144. (2)145.145.145.145.145. (1) 146.146.146.146.146. (3) 147.147.147.147.147. (4) 148.148.148.148.148. (2)149.149.149.149.149. (3) 150.150.150.150.150. (2) 151.151.151.151.151. (3) 152.152.152.152.152. (1)153.153.153.153.153. (4) 154.154.154.154.154. (1) 155.155.155.155.155. (3) 156.156.156.156.156. (4)157.157.157.157.157. (3) 158.158.158.158.158. (3) 159.159.159.159.159. (4) 160.160.160.160.160. (2)161.161.161.161.161. (1) 162.162.162.162.162. (4) 163.163.163.163.163. (3) 164.164.164.164.164. (3)165.165.165.165.165. (4)